300 Npte Questions And Answers Ptmasud.pdf

  • Uploaded by: Manik Mishra
  • 0
  • 0
  • January 2021
  • PDF

This document was uploaded by user and they confirmed that they have the permission to share it. If you are author or own the copyright of this book, please report to us by using this DMCA report form. Report DMCA


Overview

Download & View 300 Npte Questions And Answers Ptmasud.pdf as PDF for free.

More details

  • Words: 37,613
  • Pages: 921
Loading documents preview...
300 NPTE Questions and Answers By Peter Stringer Minute Help Press www.minutehelp.com © 2011. All Rights Reserved.

Disclaimer This test prep book is not endorsed by any of the administrating bodies of the NPTE exam. Examination questions can vary; it is always best practice to contact your school / testing site to ask what questions you should be prepared for.

Clinical Application of Foundation Science A physical therapist is gross manual testing the trunk of a female patient who has a history of stroke. The patient, in prone position, moves into the direction of trunk extension with her hands behind the ears. Trunk extension includes the following muscles except: A. Interspinales

B. Erector spinae C. Rectus abdominis D. Semispinalis thoracis

Answer: C – Rectus abdominis is a trunk flexor. Interspinales, erector spinae, semispinalis thoracis, intertransversarii, and multifidi, are classified as trunk extensors. A patient who is able to perform trunk extension with the hands behind the ears is a normal result. A poor result is characterized by moving into the direction of trunk extension as the arms follow.

The ankle and the foot are complex structures that work together during gait to provide balance as the body walks over an uneven ground. The ankle joint is responsible for the following: I. Lateral adjustments II. Stability III. Forward motion IV. Medial adjustments

A. I, IV B. II, III C. I, II, III D, I, III, IV

Answer: B – The ankle joint is responsible for forward motion and stability. The foot joints are responsible for medial and lateral adjustments. At heel strike, the initial contact occurs at the posterior-lateral heel. At midstance, the weight of the body is behind the metatarsal heads. At toe off, the weight is mainly over the first and second metatarsal heads.

A patient is referred to the clinic for a cardiac rehabilitation program. The history of the patient reveals a diagnosis of myocardial infarction, involving the left coronary artery. This artery divides into two major arteries: the left anterior descending artery and the circumflex artery. The left descending artery supplies which of the following: A. Lateral surfaces of the left ventricle.

B. Sinoatrial nodes. C. Anterior ventricular wall. D. Right atrium.

Answer: C – The left anterior descending artery supplies the anterior ventricular wall. Option A is supplied by the circumflex artery. Options B and D are supplied by the right coronary artery.

A physical therapist asks a female patient suspected of S1 nerve root compression to try walking on her toes. This test aims to check for muscle weakness. Walking on toes aims to tests which of the following muscles of the lower extremity? A. Flexor digitorum longus B. Sartorius C. Semimembranosus D. Flexor digitorum brevis



Answer: A – Walking on toes tests the flexor digitorum longus muscle. This muscle also assists in foot inversion. Sartorious flexes, laterally rotates, and abducts the hip joint. Semimembranosus flexes and medially rotates the knee joint. Flexor digitorum brevis flexes the proximal interphalangeal joints.

A patient who has had a stroke is assessed by the physical therapist before starting a course of physical therapy. On assessment, the patient demonstrates contralateral weakness, contralateral sensory loss of the toes, foot and leg, and inability to make decisions. Urinary incontinence is also noted. The artery that was most likely affected is: A. Internal carotid artery B. Vertebral artery

C. Anterior cerebral artery D. Middle cerebral artery

Answer: C – The anterior cerebral artery was most likely affected. A stroke in the internal carotid artery injury is manifested by aphasia, apraxia, and homonymous hemianopia. A stroke in the vertebral artery is characterized by numbness and weakness of the face, dysphagia, and facial pain. A stroke in middle cerebral artery is manifested by stupor, drowsiness, and global aphasia.

A single nerve root can supply more than one peripheral nerve. Because of this, if pressure is applied to one nerve root, the distribution of sensation or motor function is exhibited in more than one peripheral nerve distribution. If the L5 is compressed, which of the following is most likely reported by the patient? A. Low back pain that radiates to the upper buttock, to the back of the thigh, and to the back of the leg

B. Medial knee pain C. Hyposthesias in front of the thigh D. Hyposthesias in the fifth and lateral half of the fourth toes

Answer: A – If L5 is compressed, low back pain radiating to the upper buttock, to the back of the thigh, and to the back of the leg is most likely reported by the patient. Compression of L5 may affect the sciatic nerve. Option B is manifested by irritation of the saphenous nerve, which is supplied by L3 and L4. Option C is manifested by irritation of the femoral nerve, which is supplied by L2, L3, and L4. Option D is manifested by lateral plantar nerve, which is supplied by S1 and S2.

Assessment of a 3-month-old child reveals a palpable clunk when the left hip is reduced in and out of the acetabulum. The child is suspected of developmental dysplasia of the hip with subluxation. Which of the following statements most accurately describes a subluxation? A. It refers to antenatal dislocation of the hip. B. It describes the ability to subluxate the hip with passive

movements. C. It involves incomplete contact between the articular surface of the femoral head and the acetabulum. D. It refers to complete absence of contact between the articular surface of the femoral head and acetabulum.

Answer: C – Subluxation is the incomplete contact between the articular surface of the femoral head and acetabulum. Option A refers to teratologic dislocation. Option B refers to instability. Option D describes dislocation of the hip.

The cerebral cortex, the outer gray layer of the brain, is mainly responsible for conscious activities of the cerebrum. The cerebral cortex consists of four lobes. The Wernicke’s area for sensory and speech is located in which lobe? A. Frontal B. Occipital C. Parietal D. Temporal



Answer: D – The Wernicke’s area for sensory and speech is located in the temporal lobe. The Broca’s area for speech is located in the frontal lobe. The visual area is found in the occipital lobe. Interpretation of touch, pressure, pain, and temperature occurs in the parietal lobe.

There are 31 pairs of spinal nerves that branch off from the spinal cord. In the cervical region, the spinal nerves exit ___(1)__ the vertebra. In the thoracic region, the spinal nerves exit ___(2)___ the vertebra. In the lumbar region, the spinal nerves exit ___(3)___ the vertebra. A. 1- above; 2- above; 3- above B. 1- above; 2- below; 3- above C. 1- above; 2- below; 3- below

D. 1- below; 2- above; 3- above

Answer: C – In the cervical spine, the spinal nerves exit above the vertebra. However, the C8 exits below the C7 vertebra. The spinal nerves exit below their equivalent vertebrae. For example, the T5 spinal nerve exits through the foramen in the fifth thoracic vertebra.

The pain distribution of trigeminal neuralgia follows the sensory distribution of the fifth cranial nerve, which typically radiates to the maxillary branch. The maxillary branch is one of the fifth cranial nerve’s three branches. This branch supplies sensation to which area of the head? A. Front of the head B. Lower jaw C. Bottom lip

D. Side of the nose

Answer: D – The maxillary branch supplies the side of the nose. The front of the head is supplied by the ophthalmic branch. The lower jaw and bottom lip are supplied by the mandibular branch. The maxillary branch runs through the cheek, upper jaw, on top of the lips, teeth, and gums.

A football player who complains of burning pain in the supraclavicular area that radiates down to the arm is diagnosed with Grade II peripheral nerve injury. Grade II is classified as axonotmesis. Which of the following accurately describes axonotmesis? A. It involves axonal damage and Wallerian degeneration; the Schwann sheath is intact. B. It involves nerve function disruption due to demyelinization;

the axonal integrity is intact. C. It involves axonal and Schwann sheath damage; the nerve damage is permanent. D. It involves disruption of nerve function without axonal damage; remyelinization occurs within 3 weeks.

Answer: A – Axonotmesis involves axonal damage and Wallerian degeneration, with intact supporting structures. In this type of injury, the internal structure is preserved, so recovery is still achievable. Option B refers to grade I nerve damage, or neuropraxia. Option C refers to grade III nerve damage or neurotmesis.

A patient demonstrates loss of muscle strength in all four extremities due to a spinal cord injury. Which of the following is accurately derived from the finding? A. The injury in the spinal cord involves the first lumbar segment. B. The patient’s abdominal muscle strength is intact. C. The injury involves a segment below the first thoracic spinal nerve.

D. The injury involves one of the cervical spinal nerves.

Answer: D – A patient with tetraplegia most likely has an injury affecting the cervical spinal nerves. In addition to paralysis of the four extremities, the abdominal muscles are also affected, causing weakness in breathing and inability to cough effectively. An injury involving a thoracic segment causes paraplegia.

A 25-year-old female gave birth to a male newborn with talipes equinovarus involving the right foot. In most cases of clubfoot deformities, which of the following is most commonly involved? A. Toes B. Foot instep C. Ankle D. Lateral mallelolus

Answer: C – The ankle is most commonly involved in clubfoot deformities. The ankle is composed of seven tarsal bones, forming a group of bones called the tarsus. It consists of the talus, calcaneus, navicular, cuboid, intermediate cuneiform, medial cuneiform, and the lateral cuneiform.

The brainstem consists of three areas: midbrain, medulla oblongata, and pons. The medulla oblongata is composed of all afferent and efferent tracts. It is also the center for various physiological mechanisms. The following are controlled by the medulla oblongata except: A. Blood vessel diameter B. Breathing C. Visual reflex

D. Swallowing

Answer: C – Visual reflex is controlled by the midbrain. Options A, B, and D are controlled by the medulla oblongata. Breathing is also assisted by the pons. Sneezing, vomiting and heart rate are also controlled by the medulla oblongata.

The following muscles in the lower extremities allow adduction of the hip joint except: A. Adductor magnus B. Pectineus C. Gluteus medius D. Gracilis

Answer: C – Gluteus medius is a hip internal rotator. Adductor magnus, pectineus and gracilis are hip abductors. To test for the strength of the hip adductors, the therapist asks a patient to lie on the right side with the lower extremities and the lumbar spine straight. The therapist holds the upper leg in abduction and asks the patient to move the extremity upward from the table without flexing, rotating, and extending the hip.

Movements during ambulation are the result of several factors, including those produced by the muscles. Muscles are essential for acceleration, deceleration, and shock absorption. Which of the following is true about the muscles involved with gait kinetics? A. Acceleration is the result of the muscle’s eccentric activity. B. The greatest amount of muscle activity is required during the first

10% of the stance phase and last 10% of the swing phase. C. The times of least muscle activity are during periods of acceleration and deceleration. D. Locomotion requires continuous muscle activity as it involves movements from one location to another.

Answer: B – The greatest amount of muscle activity occurs during the first 10% of the stance phase and last 10% of the swing phase. Inactivity occurs during the midstance and the swing phase. Based on the information given, locomotion does not require continuous muscle activity. Acceleration is the result of the muscle’s concentric activity; deceleration is the result of muscle’s eccentric activity. The greatest amount of muscle activity occurs during acceleration and deceleration.

The obturator nerve, the largest nerve formed from the anterior divisions of the lumbar plexus, provides sensory innervation to the skin and fascia over the distal 2/3 of the thigh. This peripheral nerve is derived from the following nerve roots except: A. L4 B. L3 C. L2 D. L1



Answer: D – The obturator nerve is not derived from L1. It is derived from L2, L3, and L4. When any of these nerve roots is compressed, irritation of the obturator nerve may occur.

The anterior deltoid originates from the lateral margin and superior surface of the acromion. It inserts into the deltoid tuberosity of the humerus. Which of the following is the correct action of the anterior deltoid? A. It medially rotates, adducts and extends the glenohumeral joint. B. It stabilizes the head of the humerus in the glenoid fossa. C. It flexes the shoulder joint.

D. It flexes the glenohumeral joint and rotates the spine medially.

Answer: D – The anterior deltoid flexes the glenohumeral joint and rotates the spine medially. Option A is the action of the latissimus dorsi. Option B is the action of the subscapularis. Option C is the action of the biceps.

The mechanism of recovery after a stroke comes in two stages. The first stage of recovery occurs within the first three to six months. The second stage occurs after this time period. Which of the following changes are expected within the first three to six months after a stroke? A. Development of new synaptic connections. B. Neuronal network changes. C. Recovery of partially injured

neurons. D. Exposure of previously latent functional pathways.

Answer: C – Recovery of partially damaged ischemic neurons occurs within the first three to six months after a stroke. Resolution of local edema, augmentation of local circulation, and the destruction of local toxins are expected within the first six months of stroke. The second stage of recovery involves neuroplasticity, which includes changes of structural and functional neuron organization.

A patient who complains of fatigue, difficulty swallowing, and weakness is diagnosed with myasthenia gravis. The physical therapist, who is teaching the patient with various conservation methods, understands that this condition is most likely caused by which of the following? A. Insufficient secretion of acetylcholine. B. Insufficient dopamine levels. C. Demyelinization of the neurons.

D. Inflammation of the arachnoid and pia mater of the central nervous system.

Answer: A – Myasthenia gravis is caused by insufficient of secretion of acetylcholine, which causes a defect in the transmission of signals at myoneural junctions. Parkinson’s disease is caused by insufficient secretion of dopamine levels, causing dysfunction of the extrapyramidal system. The cause of multiple sclerosis is unknown, but its development is thought to be influenced by an autoimmune disease. Multiple sclerosis symptoms are caused by demyelinization of the neurons.



A distant runner has a transient episode of complete motor paralysis with minimal sensory involvement in the left lower extremity. The patient’s history includes a recent compartment syndrome injury, which is believed to be strongly associated with the patient’s symptoms at consult. The patient’s acute nerve injury is most likely caused by which of the following mechanisms? A. Mechanical injury

B. Stretch injury C. Crush and percussion injury D. Penetrating trauma

Answer: C – Compartment syndrome is a crush and percussion injury. Compartment syndrome injuries cause increased pressure in the surrounding tissue. This pressure compresses the supply of arterial blood of the nerve, increasing the nerve’s risk for ischemic cell damage. An example of mechanical injury is tourniquet paralysis. Stretch injuries may be caused by severe blows to a nerve and traction. An example of penetrating trauma is stab wound lacerations.

Injuries heal by secondary intention, which involves large wound separations, or by primary intention, which involves smaller wound separations. Although the process of healing does not have clear-cut delineations, clinicians recognize that healing occurs in three phases: inflammation phase, proliferation phase, and remodeling phase. Which of the following occurs during the inflammation phase? A. Wound contraction

B. Increased proteoglycans C. Increased extracellular collagen synthesis D. Neutrophil migration

Answer: D – Neutrophil migration occurs during the inflammation phase. Within 5 to 6 hours of injury, neutrophils are released to remove the debris from the injured site. These cells are replaced by the monocytes and macrophages. Wound contraction occurs during the remodeling phase. Increased proteoglycans and extracellular collagen synthesis occur during the proliferation stage.

A female patient complaining of low back pain, leg pain, and weakness in the leg and foot is diagnosed with lumbar disk herniation. Lumbar disk herniation is best described by which of the following statements? A. It involves localized bulging of the disk with annular fiber damage. B. It involves injury to the corticospinal tracts. C. It involves synovial hypertrophy and chronic inflammation of the

facet joints. D. It involves narrowing of the spinal canal.

Answer: A – Lumbar disk herniation involves slipping of the intervertebral disk through annulus into the spinal. Option B describes a spinal cord injury. Option C describes arthritis. Option D refers to lumbar stenosis.

The rotator cuff is a group of muscles and tendons that depress the humeral head against the glenoid. The rotator cuff consists of the following muscles except: A. Infraspinatus B. Subscapularis C. Supraspinatus D. Serratus anterior

Answer: D – The serratus anterior stabilizes the scapula against the chest wall. The rotator cuff consists of the infraspinatus, subscapularis, supraspinatus and teres minor. The infraspinatus and teres minor muscles support external rotation. The subscapuralis allows internal rotation. The supraspinatus abducts the arm at the shoulder.

The modalities for the application of cold and heat are used in various clinical conditions. It is essential that a physical therapist take advantage of these modalities to manage and prevent certain disorders and injuries. Which of the following indicates accurate differences between the effects of heat and those of cryotherapy? A. Heat therapy decreases muscle spasm; cold therapy increases muscle spasm.

B. Heat therapy induces decreased tissue metabolism; cold therapy increases tissue metabolism. C. Heat therapy reduces spasticity caused by an upper motor neuron lesion; cold therapy decreases spasticity. D. Heat therapy sustains increased muscle contraction; cold therapy about 27°C decreases the ability of muscles to sustain contraction.

Answer: C – Heat therapy reduces spasticity, whereas cold therapy decreases spasticity caused by lesions affecting the upper motor neurons. Heat therapy and cold therapy decreases muscle spasm. Heat increases tissue metabolism; cold decreases tissue metabolism. Heat therapy decreases the ability of muscles to sustain contraction; cold therapy about 27°C increases muscle contraction.

Joints, also known as articulations, are regions where two or more bones meet. They are classified into three types: diarthrosis, amphiarthrosis, and synarthrosis. This classification is based according to their function. Which of the following joints is an amphiarthrosis? A. Shoulder joint B. Hip joint C. Vertebral joint

D. Skull sutures

Answer: C – A vertebral joint is an amphiarthrosis, or a slightly movable joint. The shoulder joint and the hip joint are diarthroses, or freely immovable joints. Skull sutures are a synarthrosis, or immovable joint.

A therapeutic exercise program usually consists of three steps that have to be followed sequentially to be effective. The first step of the program consists of exercises that aim to regain flexibility and range of motion. Although flexibility and range of motion exercises are commonly interchanged, there are technical differences to consider. Which of the following accurately describes flexibility? A. It refers to the amount of

movement possible at a given joint. B. It refers to the mobility and length to which muscles can extend. C. It is affected by strength and mobility of the joint capsule. D. It refers to the maximum force that a muscle or group of muscles can exert.

Answer: B – Flexibility refers to the mobility and length to which a muscle can extend. Inflexibility indicates that a muscle, not a joint, has impaired mobility. Options A and C refer to range of motion. Option D refers to muscle strength.

Cryotherapy has the main effect of cooling the tissues. Although the techniques of using ice or cold applications for treatment differ, the physiological responses to cryotherapy are consistent. The following are the physiological responses to cold therapy during the first 15 to 20 minutes of cold exposure except: A. Decreased tissue stiffness B. Decreased circulation

C. Decreased arthrogenic muscle inhibition D. Decreased muscle spasms

Answer: A – Cold therapy increases tissue stiffness. Cold therapy may also cause decreased temperature, tissue destruction, increased or decreased inflammation, decreased muscle spasm, and a lower metabolism.

The shoulder is the most mobile joint in the body, but because of its wide range of movements, it is one of the most commonly injured. The shoulder consists of the humerus, glenoid, scapula, clavicle, acromion, and four other joints. Which of the following is the most commonly dislocated joint in the shoulder? A. Acromioclavicular joint B. Sternoclavicular joint C. Scapulothoracic joint

D. Glenohumeral joint

Answer: D – The glenohumeral joint is the most commonly injured joint in the body. This joint consists of the capsular and ligamentous constraints that surround the musculature and the glenoid labrum. Because of the lack of bony stability, the glenohumeral joint is at risk of injuries.

Low back pain is frequently caused by lumbar disk disease, which is influenced by aging and degenerative cascade. What is the nomenclature specific to lumbar disk disease that involves breaking off of the disk fragment from the nucleus pulposus? A. Disk bulge B. Disk protrusion C. Disk sequestration D. Disk extrusion



Answer: C – Disk sequestration is the separation of the disk fragment from the nucleus pulposus. In a disk bulge, the annular fibers are intact. Protrusion of the disk involves localized bulging of the disk with damaged annular fibers. Disk extrusion involves an extended bulge with destroyed annular fibers; the disk is intact.

A female patient complains of moderate joint pain and morning stiffness lasting longer than 30 minutes. The patient has a reactive Rheumatoid factor. The patient is diagnosed with rheumatoid arthritis. This musculoskeletal disorder is best described by which of the following statements? A. It is a degenerative disorder of the joints caused by wear and tear. B. It is an age-related metabolic

disease. C. It is a chronic systemic inflammatory disease due to an immune complex disorder. D. It is a degenerative joint disease that mainly affects the weightbearing joints.

Answer: C – Rheumatoid arthritis is a chronic systemic inflammatory disease due to an immune complex disorder. In contrast to osteoarthritis, this type of arthritis mainly affects the smaller joints, such as the interphalangeal joints. Options A and D describe osteoarthritis. Option B describes osteoporosis.

Strains and sprains are among the most commonly reported musculoskeletal injuries. These injuries account for about 50% of work-related injuries. Sprains and strains are commonly interchanged, but they are two different injuries. Which of the following statements accurately describes these injuries? A. Strain involves an injury to a ligament that is surrounding a joint. B. Sprain is characterized by

increased pain with joint motion. C. Sprain causes increased pain with isometric contraction of the muscle. D. Strain can cause joint instability.

Answer: B – Sprain is characterized by increased pain with joint motion. Sprain involves a ligament injury. Because of this, sprains can cause instability of the joints. Strain is caused by a stretching injury to a muscle. It is manifested by increased pain with isometric contraction of the affected muscle.

The shoulder is able to maintain great mobility with limited stability due to the static stabilizers, which maintain congruity, and dynamic stabilizers, which create compressive force at the glenohumeral joint. Which of the following is a static stabilizer? A. Labrum B. Serratus anterior C. Supraspinatus

D. Rhomboids

Answer: A – The labrum is a static stabilizer. Static stabilizers include the bony structures, the glenohumeral ligaments, and the joint capsule. The rotator cuff muscles are the following: infraspinatus, supraspinatus, subscapularis, and teres minor. The scapular stabilizers rhomboids and serratus anterior are dynamic stabilizers.

When utilizing heat modalities, the physical therapist considers the different conductivity properties of each tissue type. Which of the following tissues has the highest thermal conductivity? A. Muscle B. Skin C. Adipose tissue D. Cortical bone

Answer: B – The skin has the highest thermal conductivity. The thermal conductivities of the tissues mentioned are the following: skin – 0.94; cortical bone – 0.80; muscle – 0.64; and adipose tissue – 0.19. Thermal conductivity is the material’s ability to conduct heat.

A female patient reveals that there are times when her hands usually fall asleep. On history taking, the patient says that things usually slip from her fingers, without her noticing. The numbness is usually severe at night. Carpal tunnel syndrome is considered. The physical therapist understands that this condition is caused by a damaged: A. Radial nerve B. Ulnar nerve

C. Median nerve D. Axillary nerve

Answer: C – Carpal tunnel syndrome is caused by a damaged median nerve. This nerve runs through the carpal tunnel. High pressure within the carpal tunnel can damage the median nerve. This condition is more common in females. Whites have the highest risk of developing CTS.

Exercise is an essential part of anterior shoulder dislocation rehabilitation program as it can have a positive effect on muscular tissue and muscular strength. A physical therapist gradually initiates a group of exercises that involve constant velocity of shoulder movement with variable resistance. The resistance is given throughout the muscle action to allow maximum tension. This type of exercise is classified as: A. Isokinetic

B. Isometric C. Isotonic D. Endurance exercise

Answer: A – Isokinetic exercises involves constant velocity of movement with variable resistance employed throughout the muscle action. This type of exercise allows maximum tension throughout the range of motion to encourage motor recruitment. Isometric exercise involves constant joint position given in variable resistance without changing muscle length. Isotonic exercise is composed of a dynamic exercise that combines constant load with uncontrolled speed of movement. Endurance exercise

increases a muscle’s ability to maintain or repeat a contraction within a period of time.

Formation of thrombus and its manifestations are related to an inflammatory process. When thrombus develops, inflammation occurs. The inflammatory process causes the thickening of the vein wall. Which of the following types of venous thrombosis is associated with vein inflammation caused by an invasive procedure? A. Varicose veins B. Thrombophlebitis

C. Phlebitis D. Phelothrombus

Answer: C – Phlebitis involves vein inflammation caused by an invasive procedure, such as insertion of intravenous lines. Varicose veins are manifested by distention and protrusion of veins. Thrombophlebitis is a thrombus associated with inflammation. Phelothrombus is a thrombus without an inflammation.

Most cases of lumbar disk herniation are associated with the natural aging of the spine. Experts believe that disk herniation is mainly caused by the drying out and weakening of the intervertebral disks. The following factors increase an individual’s risk for disk herniation except: A. Frequent heavy lifting using the back muscles. B. Constant spinal rotation, bending, or pulling.

C. Female gender. D. Being overweight.

Answer: C – Males, especially between 30 and 50 years, are more likely to develop a herniated disk. Frequent heaving lifting using the back muscles, constant spinal rotation, bending, or pulling, and being overweight may increase the risk for this condition. Other factors that may increase the risk include sedentary lifestyle, frequent driving, and smoking.

Brunnstrom classified the stages of motor recovery into six stages. On assessment, relative independence of the limb synergies and waning spasticity are noted on the patient’s affected side. Arm raising to a side horizontal position and bringing of hand over the head are also recorded. These findings indicate what stage of motor recovery? A. 2 B. 3

C. 4 D. 5

Answer: D – The findings indicate stage 5 of motor recovery. Stage 2 is characterized by the development of basic limb synergy with no voluntary movements. Stage 3 is characterized by marked spasticity. Stage 4 is manifested by decreasing spasticity with absent sign of relative independence of limb synergies. Stage 6 is characterized by isolated joint movements.

A physical therapist is assessing the skin of an elderly patient who is recently referred for physical therapy. Understanding the normal age-related changes of the integumentary system, an elderly patient is at risk of the following conditions except: A. Extravasations of blood into the skin and mucus membranes after a minor injury. B. Heat exhaustion

C. Skin infections D. Increased wound healing rate

Answer: D - Wound healing rate decreases with aging. In addition to thinner, less protected and inelastic skin with decreased layer of fat, blood vessel changes and lowered immunity may slow down the process of healing. Wounds in the elderly may heal 4 to 5 times more slowly than in younger individuals. When healing is delayed, the risk for pressure ulcers or infections increases. The elderly are also at risk for easy bruising, mainly due to thinner, inelastic and reduced fat tissues. The number of sweat glands

decreases with aging, causing the elderly to sweat less. This change makes the elderly inefficient in regulating their body temperature in hot weather.

A 55-year-old non-athletic and healthy patient is referred for an exercise program. For beginners, a good target heart rate should be 60% of the maximum heart rate. What is the correct target heart rate during exercise for the patient? A. 165 B. 175 C. 105 D. 99



Answer: D – The target heart rate for the patient during exercise is 99. The target heart rate is obtained by using the following formula: a. 220 – age = 220 – 55 = 165 b. 165 x 0.60 = 99

Therapeutic cold, also known as cryotherapy, has the main effect of cooling the tissues. Cold application may use various techniques and duration of therapy, but it has the same basic physiological effects. The effects of cryotherapy are the following except: A. Analgesia B. Vasoconstriction C. Increased muscle efficiency

D. Decreased local metabolism

Answer: C – Cold therapy decreases muscle efficiency. In addition to analgesia, vasoconstriction and decreased local metabolism, reactive hyperemia, reduce swelling or edema, hemorrhage reduction, and reduced muscle spasm are also therapeutic effects of cryotherapy.

A female patient with swelling, redness, and pain in the leg is suspected of thrombophlebitis. There are three factors, called Virchow’s triad, associated with thrombophlebitis or venous thrombosis. The Virchow’s triad consists of the following except: A. Stasis of blood B. Blood vessel damage C. Occlusion

D. Coagulation of blood is increased

Answer: C – Occlusion is not a part of the Virchow’s triad. In thrombophlebitis, vessel damage stimulates the clotting cascade, which causes the aggregation of platelets at the site of damage. The aggregation is aggravated by increased blood stasis.

Compared to an individual who lives a sedentary lifestyle, a physically fit person usually demonstrates a different physiological profile. The following are the characteristic features of improved endurance except: A. Greater muscle strength B. Better adaptation of circulation and respiration to effort C. Lower blood pressure on exercise

D. Lower pulse rate on exercise

Answer: A – Greater muscle strength is a characteristic feature of physically fit individual, but it is not a characteristic feature of improved endurance. Options B, C, and D are characteristic features of improved endurance.

A female patient is about to get married in three months. The patient would like to delay pregnancy at least one year after the wedding. The physical therapist is correct to inform the patient that which of the following is the most effective contraception? A. Intrauterine device B. Progestin-only pill C. Progestin-estrogen contraceptives

D. Mifepristone

Answer: C – Progestin-estrogen contraceptives are chemical contraceptives with the highest rate of success. This group of contraceptives consistently inhibits the mid-cycle surge of gonadotropin that prevents ovulation. With proper use, progestin-estrogen has a 95 to 99% user success rate. Progestinonly has 93 to 99% user success rate. Intrauterine device is next with 93 to 98%. Mifepristone is mainly used as emergency postcoital contraception.

Carpal tunnel syndrome consists of symptoms and signs that develop from the entrapment of the median nerve within the carpal tunnel. The median nerve is derived from the following nerve roots: I. C6 II. C7 III. C8 IV. T1

A. I, II, III, IV B. I, II, III C. II, III D. III, IV

Answer: A – The median nerve is derived from the C6, C7, C8, and T1 nerve roots. The ulnar nerve is derived from C7, C8, and T1. The nerve root is a part of a peripheral nerve that connects the nerve to the spinal cord.

In addition to physical changes, aging may also cause changes in vital signs, which include the blood pressure, body temperature, heart rate and breathing rate. Which of the following is an expected finding in elderly patients? A. Significant decrease in body temperature B. Increased effort in breathing C. Increased maximum heart rate

D. Decreased resting heart rate

Answer: D – The resting heart rate decreases with aging. Body temperature does not change significantly in older age. The reduced subcutaneous fat may only make it more difficult for the elderly to maintain body heat. The rate of breathing does not change significantly with aging. An elderly patient who engages in exercise achieves a lower maximum heart rate.

A female patient informs a physical therapist that she has been having problems with sexual intercourse and physical sexual responses since the spinal cord injury. The injury occurred a year-and-a-half before consult. The history of the patient reveals complete motor neuron injury affecting the S2-S5 segments. Which of the following sexual dysfunctions is most likely reported by the patient? A. Absence of vaginal lubrication

with sexual stimulation B. Involuntary tightness of the vagina C. Difficulty achieving orgasm D. Loss of sexual desire

Answer: C – Difficulty achieving an orgasm is seen in patients with complete motor neuron injury affecting the S2-S5 segments. Absence of lubrication occurs in injuries involving T10-T12. Involuntary tightness of vagina is not caused by SCI. Absence of sexual desire has many causes. It may be due to psychological problems or it may be secondary to discomforts related to certain physical conditions.

The deep veins of the legs and the pelvis are the most hospitable sites for venous thrombosis, a condition where a clot forms on the wall of a vein. Which of the following veins is least likely involved in deep vein thrombosis? A. Peroneal vein B. Popliteal vein C. Posterior tibial vein D. Great saphenous vein



Answer: D – The great saphenous vein is a common vein involved in superficial vein thrombosis. The deep veins of the calf include the peroneal vein, popliteal vein, posterior tibial vein, and femoral vein. A thrombus in the deep vein is more serious than a thrombus in a superficial vein because of the risk for pulmonary embolism.

The human body consists of 31 nerve root pairs. Each nerve root has two components: the somatic component and the visceral component. Which of the following is the function of the nerve root’s somatic portion? A. It innervates the intervertebral disks. B. It provides sensory input from the skin, fascia, and muscles. C. It innervates the blood vessels.

D. It conveys impulses to the dura mater, ligaments, and periosteum.

Answer: B – The somatic component of a nerve root provides sensory input from the skin, fascia and muscles. It also innervates the skeletal muscles. The visceral component innervates the blood vessels, dura mater, intervertebral discs, ligaments, and periosteum.

The knee is a complex structure that plays a major role in supporting the body during static and dynamic activities. It is supported by muscles, ligaments, cartilage, tendons, and bones to maintain its function. Which of the following is the strongest ligament in the knee? A. Medial collateral B. Posterior cruciate C. Lateral collateral

D. Anterior cruciate

Answer: B – Posterior cruciate ligament is the strongest ligament in the knee. It has a tensile strength of 2000 N, which is twice as strong as the ACL. The ACL is half as strong as the medial collateral ligament. The MCL is stronger than the lateral collateral ligament.

A woman at 12 weeks’ gestation is informed what to expect during labor and delivery. The patient is diagnosed with spinal cord injury affecting T11. The patient is informed that the following is more likely to occur during birth except: A. A painful birth B. Ineffective uterine contractions C. Autonomic hyperreflexia D. Abnormal progression of labor



Answer: C – Autonomic hyperreflexia concomitant uterine contractions occurs in women with SCIs above T6. SCI above T10, effective uterine contractions are normal; labor progression is expected. Afferent innervation of the uterus arises from T10 to L1, so labor is painless for women with an SCI above T10.

The screw home mechanism of the knee, or locking of the knee, occurs at the end of the knee extension to reduce the work exerted by the quadriceps during standing. Which of the following ligaments controls this mechanism? I. Medial collateral ligament II. Posterior cruciate ligament III. Anterior cruciate ligament IV. Lateral collateral ligament

A. I and IV B. II and III C. I and III B. II and IV

Answer: B – The anterior and posterior cruciate ligaments control the passive screw home mechanism. The medial and lateral collateral ligaments apply restraints to valgus and varus knee angulation, respectively.

A nerve root is the portion of a peripheral nerve root that connects the nerve to the spinal cord. The nerve roots usually arise from each level of the spinal cord. In the cervical spine, which of the following nerve roots is the most commonly compressed? A. C6 B. C7 C. C5

D. C8

Answer: A – The C6 is the most frequently affected or damaged nerve root in the cervical region because of the extensive degeneration of the C5 to C6 disc interspace. The C5 and C7 are also frequent sites of damage and compression.

The cardiovascular system and pulmonary system show the most significant changes in response to increased activity or inactivity. A patient referred for an exercise program is expected to demonstrate which of the following as a normal response to exercise? A. Decreased systolic blood pressure B. Decreased cardiac output C. Increased peripheral vascular resistance

D. Increased tidal volume

Answer: D – Increased tidal volume is a normal response to exercise. The tidal volume is the volume of air breathed in and breathed out during quiet respirations. Peripheral resistance decreases with exercises. Systolic blood pressure and cardiac output increase with exercise.

A 50-year-old female complains of cervical pain that radiates to the left anterior arm and radial side of the hand. Manual muscle testing of the biceps reveals a grade of 3. A diagnosis of cervical spondylosis is made. Which of the following is true about cervical spondylosis? A. It is strongly influenced by genetics. B. The onset and rate of progression is not influenced by repetitive,

subclinical trauma. C. In patients older than 40 years, chronic cervical degeneration is the most common cause. D. It affects the vertebral bodies and intervertebral disks of the neck; the contents of spinal canal are not usually affected.

Answer: C – Cervical spondylosis is usually caused by chronic cervical degeneration in individuals 40 years and above. It affects the vertebral bodies, intervertebral disks, nerve roots, and spinal cords. It is not clear whether genetics play a role in cervical spondylosis development. The onset and rate of progression may be influenced by repetitive, subclinical trauma. Individuals who carry loads on their heads repeatedly are more likely to develop cervical spondylosis.

A physical therapist is assessing the respirations of a patient referred for therapy due to adhesive capsulitis. Physical examination findings suggest that the patient has a normal respiratory function. In patients with normal respiratory function, which of the following occurs during inspiration? A. The pressure in the alveoli is greater than the atmospheric pressure.

B. The intrathoracic volume increases. C. The ribcage descends and contracts. D. The diaphragm descends.

Answer: D – During inspiration, the diaphragm descends. The elongated mediastinal structures and compressed abdominal contents are pulled upward on the diaphragm. Options A, B, and C occur during expiration.

A female patient with angina is referred to the clinic after a total knee replacement. The patient has a history of angina, which is usually relieved by rest and intake of nitroglycerin. The pain occurs with increased activity or stress. Based on these details of information, the patient has which type of angina? A. Unstable angina B. Variant angina C. Prinzmetal angina

D. Stable angina

Answer: D – The patient has stable angina. This type of angina is related to myocardial demand. Option A is manifested by angina at rest. Options B and C refer to the same type of angina; this is related to coronary artery vasospasm.

A physical therapist reviews the medication list of a patient who is maintained on bronchodilators for the management of bronchial asthma. Which of the following is a common side effect of bronchodilators? A. Hypertension B. Sedation C. Tachycardia D. Muscle spasms

Answer: C – Bronchodilators may cause tachycardia. Other side effects of bronchodilator therapy include dizziness, chest pain, hypotension, and muscle weakness. Hypertension is usually caused by decongestants. Antitussives may cause sedation.

At higher altitudes, a patient experiences dizziness, nausea, weakness, decreased vision, and headache. The physical therapist informs the patient that these symptoms are mainly caused by lowered partial pressure of ambient oxygen at higher altitudes. Which of the following is a long-term response to decreased partial pressure of ambient oxygen? A. Increased stroke volume

B. Increased aerobic power C. Increased number of red blood cells D. Decreased amount of hemoglobin

Answer: C – Increased number of red blood cells is a long-term response to decreased partial pressure of ambient oxygen. Other long-term responses to altitude include increased stroke volume and decreased aerobic power. The amount of hemoglobin and the number of RBCs increase to maintain the actual oxygen-carrying capacity of the blood within sea-level capacity, in spite of the reduced saturation of hemoglobin.

A mother asks a physical therapist when she can start toilet training her 18-month-old child. The therapist informs the mother that each child develops at his or her own phase, so a definite answer is not possible. In spite of this, there are signs indicating readiness for toilet training. The following suggest readiness for toilet training except: A. Child is able to stay dry for two hours.

B. Child is able to sit, squat and walk. C. Child is able to sit on the toilet for 5 to 10 minutes without getting off. D. Child’s bladder control develops; bowel control insignificant.

Answer: D – A child usually achieves bowel control before bladder control. Options A, B, and C are indicative of readiness for toilet training. Other signs of readiness include an ability to get up dry from a nap, and the ability to remove clothing.

A patient with chronic obstructive pulmonary disease is assessed by a physical therapist for an exercise program. Pulmonary assessment is conducted through physical examination and pulmonary function tests. A patient with chronic obstructive disease is expected to have which of the following findings? A. Decreased residual volume B. Increased forced vital capacity

C. Increased total lung capacity D. Increased forced expiratory flow

Answer: C – A patient with increased total lung capacity is commonly assessed with increased total lung capacity. The residual volume is expected to increase. The forced vital capacity and forced expiratory flow decrease because of narrowed airways.

Equipment and Devices and Therapeutic Modalities

The healing process is subtly or significantly influenced by external factors, which include various treatment modalities. The application of electrical stimulation has the following effects during rehabilitation except: A. Reduced tissue viscosity B. Enhanced protein synthesis C. Relaxed muscle spasm D. Re-established lymphatic flow



Answer: A – Reduced tissue viscosity is mainly achieved through thermal therapy. Tissue viscosity is reduced to make an area more supple or pliable for stretching. Electrical stimulation relaxes muscle spasm and relieves pain. By stimulating contractions, lymphatic fluid is pumped into the lymphatic system, relieving edema and pain. By enhancing protein synthesis, electrical stimulation promotes healing.

Superficial heating modalities use the subcutaneous fat tissue to act as a thermal insulator and inhibit the heat transfer. Heat transfer is classified into three types: conduction, convection, and conversion. If a physical therapist decides to use a modality that uses the convection mechanism of heat exchange, which of the following is most likely considered? A. Electric heating pad

B. Whirl pool C. Warm compresses D. Ultrasound

Answer: B – Whirlpool is a modality that involves heating through convection mechanism. Fluidotherapy, moist air baths, and whirlpool involve the production of heat by movement of the transferring medium. Electric heating pad and warm compresses are examples of conductive heating, where the heat is transferred from one point to another without movement of the medium. Ultrasound makes use of conversion, which involves heat exchange by the conversion of one energy form to another.



Which of the following is considered as the most effective orthosis for use in controlling rotation and lateral bending at C1-C3? A. Sternal-occipital-mandibular orthosis B. Halo device C. Philadelphia collar D. 4-poster brace

Answer: B – The halo device is the best orthosis for controlling rotation and lateral bending at C1-C3. The halo device is also the most effective for use in controlling flexion and extension at C1-C3. The sternaloccipital-mandibular orthosis controls extension less effectively than the other orthoses.

A 30-year-old female patient with chronic lower back pain secondary to lumbar spinal stenosis is ordered to be placed on lumbar traction. Which of the following actions by the physical therapist demonstrates the correct technique of applying this type of traction? A. The patient is placed in seated or supine position. B. The pelvic belt is hooked around the patient; the top edge is around the

iliac crest. C. The thoracic belt is hooked around the patient; the inferior margin is slightly above the lower ribs. D. A force ¼ to ½ of the patient’s lower extremity is initially placed.

Answer: B – The pelvic belt is hooked around the patient with the top edge around the iliac crest. The belt aims to deliver a caudal pull. The thoracic belt is hooked around the patient with the inferior margin slightly below the lower limbs to deliver a cephalad pull. The patient should be in supine position. Initially, low weights are used.

Upper-limb orthoses are classified according to their function. Which of the following upper-limb orthoses augments prehension with more proximal joint movements? A. Assistive B. Substitutive C. Protective D. Corrective

Answer: B – Substitutive upper-limb orthoses are used to augment prehension with more proximal joint movements. Assistive upper-limb orthoses are used for patients with neuromuscular weakness to place an extremity in functional position. Protective upper-limb orthoses are used to prevent contractures. Corrective upper-limb orthoses are used to increase range of motion.

A physical therapist is observing a patient who is walking with crutches. The patient advances both crutches forward, and then swings both legs past the crutches at the same time. Based on the sequence, the patient is demonstrating which crutch gait? A. Two-point crutch gait B. Swing-to crutch gait C. Swing-through crutch gait D. Tripod crutch gait



Answer: C – The patient is walking with crutches using the swingthrough crutch gait. This technique is recommended to patients who are unable to fully bear weight on both legs. Option A is indicated to patients with weakness in both legs. Option B swings both legs at the same time without going past the crutches. Option D is indicated to patients with paraplegia taught to do swing to gait pattern.

Superficial heating modalities are usually used to increase cutaneous blood flow and to provide cooling reactions as they remove the heat that is applied externally. A physical therapist uses a superficial heating modality that involves conversion of one energy form into another. Which of the following heating modalities is most likely used? I. Fluidotherapy II. Paraffin bath III. Heat lamp

IV. Radiant light A. I and II B. II and III C. III and IV D. I, III, IV

Answer: C – Superficial heat modalities that use conversion heating include the heat lamp and radiant light. These modalities involve production of superficial heat, with the heat being transferred when the conveying medium is converted to heat energy at the surface of the skin. Fluidotherapy uses convective heating. Paraffin bath uses conductive heating.

A patient is ordered to ambulate with assistance using a pair of crutches. The physical therapist instructs the patient to move the right crutch ahead, and then move the left foot forward at about the level of the right crutch. The next instruction is to move the left crutch forward. The right foot is moved forward last. Which of the following is accurately derived from the instructed gait sequence? A. This gait sequence is the least

stable crutch gait pattern. B. This gait sequence involves some weight bearing on both legs. C. This gait sequence is contraindicated in patients with ataxia. D. This gait sequence necessitates good trunk balance and good strength in the upper limbs.

Answer: B – A four-point crutch gait pattern involves some weight bearing on both legs. It is a stable gait because three points are consistently in contact with the floor. The fourpoint crutch gait is recommended to patients with ataxia. The swingthrough gait needs good trunk balance and strong upper limbs.

Transcutaneous electrical nerve stimulation is one of the most commonly used forms of electroanalgesia. Which of the following therapeutic methods of TENS is the most recommended for acupuncture-like settings? A. High-stimulation frequency and low-intensity, just above the threshold; current is set between 10 and 30 mA. B. Low-stimulation frequency and

high-stimulus intensity, close to the tolerance level of the patient. C. Low-intensity stimuli firing in high-frequency bursts. D. Frequency of each burst is 1 to 2 Hz and the frequency of impulses with each burst is 100 Hz.

Answer: B – In acupuncture-like settings, the therapeutic method of TENS usually involves lowfrequency stimulus from 1 to 10 Hz, at high stimulus intensity. Option A describes the conventional TENS. Options C and D refer to pulsed TENS.

The modalities of deep heating convert energy into heat as it goes through body tissues. Electromagnetic radiation, highfrequency sound and high-frequency currents are some of the energy sources used. Which of the following heating modalities is considered as the best method for large-area deep heating? A. Microwave diathermy B. Hydrotherapy

C. Ultrasonography D. Shortwave diathermy

Answer: D – Shortwave diathermy is the best modality for large-area deep heating. This modality is used to treat localized musculoskeletal pain, joint inflammation, pain or spasm, sprains or strains, and periostitis. Hydrotherapy is a superficial heating modality. Ultrasonography is the best method for deep heating large joints. Microwave diathermy can selectively heat muscles.

Proper crutch fitting is essential as it facilitates proper healing, promotes stability when walking, and prevents complications and injuries. The following actions ensure proper crutch fitting except: A. The lengths of the crutches are adjusted, ensuring 1 to 2 inches of space between the axillae and the top of the axillary pads when standing. B. The hand grip is adjusted, allowing the elbow to flex to about

30-degrees when the patient is standing up straight. C. The handgrips of the crutches are even with the top of the hip line. D. Ensure that the patient does not lift himself/herself when pushing down and extending the arms.

Answer: D – The position is correct if the patient is able to lift herself slightly off the floor when pushing down and extending the arms. In this case, the lengths of the crutches require adjustment. The crutches should not fit into the axillae. The weight of the body must be supported almost entirely by the hands.

Therapeutic methods of transcutaneous electrical nerve stimulation application can use one of the three options of standard settings. Which of the following standard settings is the most appropriate for conventional TENS? A. High-frequency stimulus from 40 to 150 Hz, at low stimulus intensity with the current set from 10 to 30 Hz. B. Low-frequency stimulus from 1 to

10 Hz, at high intensity stimulus. C. High-frequency bursts, at low intensity stimulus; the frequency of impulses within each burst is 100 Hz. D. Low-frequency stimulus at low intensity stimulus.

Answer: A – Conventional TENS unit delivers high-frequency stimulus from 40 to 150 Hz at low stimulus intensity with the current set 10 to 30 Hz. Option B is the standard setting in acupuncture-like settings. Option C is the standard setting of pulsed TENS units.

A male elderly who had a recent total knee replacement is ordered an assistive device that aims to help him with his balance and walking. Which of the following ambulatory assistive devices can give the patient the most stability? A. Quad cane B. Walker, standard C. Forearm crutches D. Knee-support crutches



Answer: B – A walker with four solid prongs gives the most stability. This ambulatory assistive device is considered for patients who had a total knee replacement or have a significant impairment with balance and walking. Walkers are usually recommended to patients with good upper-limb strength.

A patient sustained full-thickness in the left anterior arm, including the anterior and posterior axillary regions. An orthosis is ordered to the patient to be worn before and after skin grafting. Which of the following orthoses is the most appropriate for the patient? A. Figure-8 harness B. Overhead sling suspension C. Airplane splint

D. Balanced forearm orthosis

Answer: C – A shoulder-elbow-wrist orthosis, an airplane splint is often used in patients with axillary burns. This orthosis provides maximum contact as it keeps the glenohumeral joint in maximum abduction. It is mainly used to decrease the risk of contractures. Option A is used to restrict motion in clavicular fractures. Option B is used in patients with proximal arm weakness. Option D is ordered for patients with high-level tetraplegia.

An infant is diagnosed with correctible talipes equinovarus involving the right foot. The physical therapist expects the following corrective interventions for the patient except: A. Manipulation B. Casting C. Splintage D. Cincinnati incision

Answer: D – An easy or correctible clubfoot deformity is managed with manipulation, casting, and splintage. Surgery is indicated to resistant clubfoot deformities or to deformities that fail to respond to non-operative interventions. Options for incisions include the Cincinnati incision and Turco curvilineal medial or posteromedial incision.

Because of its ability to treat or relieve pain efficiently with minimal side effects, a variety of newer transcutaneous electrical stimulation modalities have emerged. Percutaneous electrical nerve stimulation or PENS is one of these. Which of the following accurately describes this modality? A. It is based on summation of two alternating current signals that come in slightly different frequencies.

B. When the signals are in phase, they summate to an amplitude sufficient to cause stimulation. C. It uses two, four, or six applicators, delivering higher currents than TENS. D. It uses acupuncture-like needle probes to deliver electrical stimuli at the precise level.

Answer: D – Percutaneous electrical nerve stimulation uses acupuncturelike needle probes as electrodes to deliver stimuli at a precise level in close proximity to the nerve endings. Options A, B, and C describe the interferential current therapy.

Orthoses are usually classified into two types: static and dynamic. The classification is mainly based on the amount of joint movement that each orthosis allows. Which of the following is not correct about static orthoses? A. Static orthoses do not allow joint movements. B. They are mainly ordered for patients with weakened muscles. C. They function as rigid supports in

fractures. D. They are used for promoting eventual joint movement.

Answer: B – Dynamic, not static, orthoses are ordered for patients with weakened muscles or limited control of the neuromuscular system. In contrast to dynamic orthoses, static orthoses do not allow joint movement. This type serves as a rigid support in fractures, inflamed tendons and soft tissue, and nerve injuries. They are also used to promote eventual joint movement through serial or progressive orthoses.

A baseball player with acute medial epicondylitis is referred for physical therapy. The patient record includes a history of elbow pain during sleep. Which of the following orthoses is the most recommended for the patient? A. Counterforce bracing B. Cock-up wrist splint C. Wrist extension splint D. Ulnar gutter splint



Answer: B – The cock-up wrist splint is the most recommended orthosis for the patient. This splint is especially useful for athletes who complain of wrist pain during sleep because it maintains the extremity in neutral position. Option A is the most recommended during activities as it decreases the contraction forces of the flexor-pronator muscles.

A 20-year-old male complains of pain in the ball of the left foot, causing limited movements. On assessment, the patient’s second toe is bent upward from the left foot’s ball joint. Corns on the top of the involved toe are also noted. Which of the following is the most recommended footwear for a patient with claw toes? A. Moccasin shoe B. Oxford shoe

C. A shoe with a high upper D. A shoe with a Blucher throat style

Answer: A – Moccasin shoe is the most recommended footwear for the patient. Individuals with claw toes are recommended to wear shoes with a spacious toe box, such as moccasin shoes. An oxford shoe has a low upper that comes with a laced closure. A shoe with a high upper is recommended for patients wearing an ankle-foot orthosis. A shoe with a Blucher throat style allows the patient to fold the shoe’s tongue back to create a large opening into the shoe.

A stroke physical therapy program aims to achieve normalization of muscle tone to encourage normal movements. Which of the following therapy modalities is correctly included in the program as an effective tone-reducing intervention? A. Warm compresses B. Pressure splints C. Stretching of finger muscles through manual stretching techniques

D. Passive manipulation by the therapist

Answer: D – Passive manipulation by the therapist is an effective tonereducing technique. Icing, contraction of muscles antagonistic to spastic muscles, and weight bearing are the other techniques used to reduce muscle tone. Pressure splints and manual stretching of finger muscles do not cause longterm effects.

A physical therapist includes transcutaneous electrical nerve stimulation application as one of the modalities of treatment for a patient complaining of pain after a spinal cord injury. The physical therapist is applying the TENS machine correctly by demonstrating which of the following actions? A. The machine is switched on before placing the electrode pads on the skin.

B. The therapist places self-adhesive pads within an inch of each other. C. The pulse rate is set before switching on the machine. D. The pads are placed over the area of sensory impairment.

Answer: C – The physical therapist sets the pulse rate before switching on the machine. The machine must be switched off before placing the electrode pads on the skin. The pads should not be applied within an inch of each other. The pads should be placed on the skin over painful areas. They should not be placed over the area of sensory impairment as it increases the risk of burns.

A patient with chronic fatigue syndrome is indicated to have a myofascial release, a form of therapy that stretches the fascia. This therapy relieves the patient’s discomforts by which of the following mechanisms? A. It involves placing a joint or body part into a position of maximal ease, allowing the shortened muscles to reset their spindles. B. It involves positional techniques that change inappropriate physical

alterations of neural tissues in response to stimuli. C. It involves stretching of tight areas to equalize muscle tension throughout the body. D. It encourages proprioceptive neuromuscular facilitation through isometrics.

Answer: C – Myofascial release involves stretching of tight areas to equalize muscle tension throughout the body. When muscle tension is unequal, compression of nerves and muscles can cause pain. Option A is counterstrain. Option B describes indirect positional techniques. Option D is muscle energy.

Patients with heel spurs, Achilles’ tendon contracture, and hindfoot malalignment are managed by shoe modifications to provide support during gait and to reduce pain. Which of the following shoe modifications is the most likely considered for these patients? A. External heel wedge B. Internal scaphoid pad

C. Internal heel cushion relief D. External heel elevation

Answer: C – Internal heel cushion relief manages the conditions mentioned. Orthotic interventions for these patients include internal heel modifications. An internal scaphoid pad is used for medial arch support.

There are different of types of traction utilized in clinical practice. Which type of traction is used if the tractive force applied to the patient is provided by a motorized or hydraulic pulley system that comes with weights attached through sling devices? A. Mechanical traction B. Manual traction C. Autotraction

D. Gravitational traction

Answer: A – Mechanical traction is the use of a motorized or hydraulic pulley system. Manual traction involves the use of the therapist’s hands and body weight in providing the tractive force. Autotraction is controlled by the patient by pulling on bars at the head of the table without assistance. Gravitational traction uses a tilt table and underwater variations of traction.

Knee-ankle-foot orthothics or KAFOs consist of an ankle-foot orthosis with metal uprights, two thigh bands and a mechanical knee joint. KAFO is indicated to the following patients except: A. A patient with quadriceps paralysis B. A patient with genu valgrum C. A patient with a patellofemoral disorder

D. A patient ordered to limit weight bearing of the thigh, leg and foot with quadrilateral containment brim

Answer: C – Patellofemoral disorders are managed by knee orthoses only. KAFO is usually made of metal-leather and metalplastic or plastic and plastic metal. Plastic designs are indicated for maximum foot control and for closer fit.

In the cervical spine, traction is used for elongation. With the cervical region of the spine flexed at 30degrees, the most favorable weight to accomplish cervical separation is: A. 100 pounds B. 300 pounds C. 25 pounds D. 20 pounds

Answer: C – The most favorable weight to accomplish cervical separation is 25 pounds. Widening of lumbar interspaces requires 70 to 300 pounds of pull. Application of 300 pounds manually can increase 1 cm interspace distance in the lumbar spine.

A female runner sustained a stress fracture involving the inferior aspect of the left femoral neck. A treatment regimen, including protection with the use of crutches, is prescribed. The physician orders non-weight bearing ambulation until relief of pain at rest is achieved. A physical therapist instructs the patient to walk with crutches using which of the following crutch gaits? A. Four-point crutch gait

B. Three-point crutch gait C. Two-point crutch gait D. Swing-to crutch gait

Answer: B – Three-point crutch gait is recommended to patients who are instructed not to bear weight on one leg. Patients with fractures, pain, and amputations involving one of the lower extremities are taught to first move the crutches and the affected leg forward, and then bear weight down through the pair of crutches. The stronger leg moves forward next. Options A and C are recommended to patients with weakness in both legs. The two-point crutch gait is faster than the four-point crutch gait. Option D is also recommended to

patients with weakness of both lower extremities; this pattern necessitates good upper extremity strength.

The major physiologic effect of traction in the lumbar spine is elongation. The optimum weight for lumbar traction to accomplish widening of lumbar interspaces is between: A. 25 and 100 pounds B. 70 and 300 pounds C. 50 and 100 pounds D. 90 and 250 pounds

Answer: B – The optimum weight to accomplish widening of lumbar interspaces with the use of traction is 70 to 300 pounds. Cervical elongation is achieved with the addition of 25 pounds with a cervical flexion of 30-degrees.

A female patient with a chronic pelvic inflammatory disease is ordered to complete a course of therapy that consists of various modalities, including deep heat treatment. The patient is ordered to have a deep modality that involves application of high-radiofrequency electrical currents administered at 27 MHz. The patient is ordered which of the following modalities? A. Shortwave diathermy

B. Ultrasonography C. Microwave diathermy D. Radiant heat

Answer: A – Shortwave diathermy involves application of highradiofrequency electrical currents. Ultrasonography uses highfrequency acoustic vibrations. Microwave diathermy utilizes a form of electromagnetic radiation.

A female patient with a cervical disc disorder is ordered to have a cervical sliding device, a form of cervical traction. Which of the following demonstrates the correct technique of applying this type of traction? A. The patient’s head is placed against a padded headrest with the neck extended. B. The patient’s head is attached to a system that provides a pull in a cephalad direction.

C. Initially start with a force from 20 to 35 pounds. D. The device is placed on the back of the skull and under the mandible, and the straps are secured to ensure that the force is applied to the occipital area.

Answer: B – The patient’s head should be attached to a system that provides a pull in a cephalad direction. Cephalad is going toward the head. The head of the patient should be placed against the headrest with 20- to 30-degrees of cervical flexion. The treatment must start with a low force of 10 to 15 pounds, gradually increasing to 20 to 35 pounds. A cervical halter is placed on the back of the skull and under the mandible. Cervical sliding device involves securing a head strap across the forehead and a beam under the

mastoid process.

A patient with pneumonia is managed by postural drainage, which allows removal of secretions from a specific lung segment through the aid of gravity. Physical examination suggests that the secretions are pooled in the lower lobes and anterior basal segments. Which of the following actions can effectively drain the secretions? A. The foot of the bed elevated to 16 inches; the patient is placed with the head down and in left side-lying

position. B. The foot of the bed is elevated to 20 inches; the patient is placed with the head down and in prone position. C. The foot of the bed is elevated to 20 inches; the patient is placed with the head down, in side-lying position. D. The bed is maintained flat; the patient is placed in prone position with two pillows under the hips.

Answer: C – Draining the lower lobes and anterior basal segments requires the foot of the bed to be elevated to 20 inches with the patient placed head down in side-lying position. Option A drains the right middle lobe. Option B drains the lower lobes and lateral basal segments. Option D drains the lower lobes and superior segments.

A physical therapist is treating a patient with inability to extend the cervical spine more than 30% of its normal range due to central cervical pain at the C5-C6 level. The patient has been complaining of cervical pain for the past three days. A positional release is performed by the therapist. Which of the following is the most appropriate method in opening the affected foramen? A. Forward flexed, sidebent contralaterally and rotated

contralaterally B. Forward flexed, sidebent ipsilaterally, and rotated contralaterally C. Forward flexed, sidebent contralaterally, and partially rotated contralaterally D. Forward flexed, sidebent ipsilaterally, and partially rotated contralaterally

Answer: A – The neck is forward flexed, sidebent contralaterally and rotated contralaterally. This position allows the contraction of the posterior fibers of the upper trapezius.

Mechanical ventilators come in different types. Which of the following ventilators is the most responsive to patients who have a severe lung disease or require prolonged weaning?? A. Time-cycled ventilator B. Pressure-cycle ventilator C. Volume-cycle ventilator D. Microprocessor-cycle ventilator

Answer: D – Microprocessor-cycle ventilator is the most responsive to patients who have severe lung disease or require prolonged weaning. A microprocessor is built into a ventilator to allow continuous monitoring of ventilator functions and patient parameters.

Range-of-motion exercises are exercises that move a joint through the degree of its limitations. Which type of range-of-motion exercises involves movement of a joint done by the physical therapist or a mechanical device without any muscle contraction exerted by the patient? A. Active-assistive range of motion exercise B. Active range of motion exercise

C. Passive range of motion exercise D. A and C Answer: C – Passive range of motion exercises involve movement of a joint done by a physical therapist or a mechanical device without requiring muscle contraction by the patient. Active range-of-motion exercises involve active movements by the patient; this type of ROM is only used when the patient is able to voluntarily contract the muscle of a

given joint without any restriction. Active assisted range-of- motion exercises is a type of ROM that requires assistance; the patient is able to assist in the motion but cannot complete the motion independently.

Stretching exercises are exercises that improve a joint’s range of motion. Which of the following stretching exercises involve forceful, rapid, and intermittent stretches with high speed and high intensity? A. Self-stretching exercises B. Manual passive stretching C. Ballistic stretching D. Mechanical resistance

Answer: C – Ballistic stretching is a stretching exercise that uses forceful, rapid, and intermittent stretches. This type is usually indicated to athletes and younger individuals who are receiving rehabilitation. Selfstretching exercises are stretching exercises that a patient can perform independently. Manual passive stretching is the manual application of an external force to move a specific body segment slightly above the point of resistance and available range of motion.

Open-chain exercises or closed-chain exercises are usually part of the exercise prescription given to a patient who is referred for exercise therapy. Which of the following activities is classified as an openchain exercise? A. Bicep curls B. Squats C. Push-ups D. Leg press



Answer: A – Biceps curls is classified as an open-chain exercise. Open-chain exercises involve movements in which the distal body part, such as a hand or a foot, is allowed to move freely. Options B, C, and D are closed-chain exercises. Closed-chain exercises involve movements in which the distal part is fixed, with the proximal body parts moving around it.



Examination

A physical therapist is manual muscle testing the left infraspinatus muscle of a patient who is suspected of rotator cuff disease. Which of the following positions best isolates the infraspinatus muscle to test strength? A. Arm is placed to the side; resist external rotation. B. Arm is placed in neutral position and with the thumb up; resist abduction. C. Arm is maximally internally

rotated up the back with the back of the hand against the back; lift the arm off the back. D. Arm is placed up the back, at least the sacrum level, with the back of the hand facing the patient’s arm, and the elbow is flexed to 90-degrees; arm is held at that position.

Answer: A – To strength test the infraspinatus, the patient’s arm is placed to the side. The patient is asked to resist external rotation. If the patient cannot resist external rotation, infraspinatus pathology is considered. Option B isolates the supraspinatus muscle. Option C describes the lift-off sign, which tests the subscapularis muscle. Option D tests for subscapularis lag sign.

A physical therapist conducts a gait assessment and balance evaluation test on a patient. Which of the following findings obtained during the test demonstrates an abnormal gait pattern, requiring referral for further evaluation? A. At foot contact, the dorsiflexors eccentrically contract and the foot plantar flexes from heel strike to a foot-flat position. B. The hip joint moves from 20 to

30-degrees of flexion at initial contact to 10-degrees of extension in terminal stance as the trunk smoothly moves over the stance limb. C. At midstance, the pelvis shifts toward the swing leg; lateral movement of the hip toward the stance leg is observed. D. On initial contact and heel strike, the knee is fully extended.

Answer: C – When the pelvis shifts toward the swing leg and the hip laterally moves toward the stance leg, the gait abnormality is termed uncompensated gluteus medius. During normal stance, the downward rotation in the frontal plane is limited by the hip abductors. The other options indicate normal patterns.

A physical therapist tests the strength of the patient’s extensor carpi ulnaris muscle by having the patient’s forearm positioned in full pronation on top of a table. The patient is instructed to extend the hand and deviate it toward the ulnar side as the therapist applies resistance against the dorsum of the fifth metacarpal bone. The therapist notes that the muscle being tested is able to move the joint against gravity, but not through a full range of motion. The muscle strength of extensor carpi ulnaris is graded:

A. 1 B. 2 C. 3 D. 4

Answer: C – The strength of the extensor carpi radialis is 3. A grade of 1 is manifested by muscle contraction without joint movement. A grade of 2 is manifested by joint movement by the muscle, but not against gravity. A grade of 4 indicates that the muscle is able to move the joint through a full range of motion but the strength of the resistance is not equal to the opposite joint.

A series of clinical tests are performed on a female newborn. The examiner evaluates the child’s hips one at a time to check for an unstable hip. The examiner starts by placing his thumb over the child’s inner thigh and index finger over the greater trochanter. The child’s hip is abducted, and gentle pressure is applied over the greater trochanter. The examiner performs which of the following tests? A. Ortolani test

B. Barlow test C. Galleazi sign D. Thomas test

Answer: A – The examiner performs the Ortolani test. A positive sign is indicated by a clunk, which is felt when the hip is reduced into the acetabulum. Barlow test is also performed to detect dislocations or subluxations in newborns, but this test is performed by applying gentle posterior pressure to the hips. Galleazi sign is performed in infants older than 3-months. Thomas test is used to evaluate for hip contractures.

A patient with low back pain demonstrates motor weakness on knee extension and sensory loss in the medial area of the foot. The physical therapist suspects L3-L4 disk herniation. Which of the following is the screening examination to localize L4 nerve root compromise? A. Walking on toes B. Heel walking C. Squat and rise

D. Thomas test

Answer: C – Squat and rise is the screening exam for localizing L4 nerve root compromise. Option A is the screening exam for S1 nerve root compromise. Option B is for L5 nerve root compromise. Option D tests for psoas tension in cases of hip flexion contracture.

A patient complains of achy pain in the lower back and sacroiliac joints. Morning stiffness is also reported. During evaluation, a physical therapist evaluates for forward flexion and lumbar mobility. Which of the following tests is used to measure the lumbar flexion? A. Schober test B. Slump test C. Hip drop test

D. Nachlas test

Answer: A – The Schober test is used to measure lumbar flexion. Patients with spondyloarthropathies usually test positive for Schober test. The slump test may detect impingement of the dura and spinal cord or nerve roots. The hip drop test is a screening tool that detects lumbar spine compensation to sacral base declination. The Nachlas test assesses irritation femoral nerve roots.

As a part of physical assessment, the examiner performs the Barlow maneuver on a 3-month-old child to check for an unstable hip. The examiner performs this maneuver by which of the following actions? A. The examiner places the child in supine position and adducts the hips; slight gentle backward pressure is applied to the head of each femur. B. The examiner places the child in supine position and examines one

hip at a time; the examiner places his thumb over the inner thigh and applies forward pressure to the femoral head. C. The examiner places the child in supine position with the hips and knees flexed. D. The examiner observes the child for any signs of painless limp.

Answer: A – Barlow test is conducted by applying backward pressure to the femoral head. Option B describes the Ortolani test. Option C refers to the Galleazi sign. Option D is conducted on older children.

A patient with a restricted motion of the left knee, secondary to pain, is suspected of meniscus injury. The McMurray test, a provocative maneuver that causes impingement, is done to aid the diagnosis. Which of the following is the correct way of performing the McMurray test? A. Patient position: seated with the knee flexed at 90-degrees; Action: the therapist internally and externally rotates the tibia.

B. Patient position: prone with the knee flexed at 90-degrees; Action: the therapist applies an axial load as the lower leg is internally and externally rotated. C. Patient position: supine; Action: the therapist extends the knee with the tibia internally rotated and applies varus stress, and then extends the knee with the tibia externally rotated and applies valgus stress D. Patient position: supine; Action: the therapist flexes the knee and medially rotates the tibia as the

patella is pressed medially with the heel of the other hand, using the fingers the palpate for the medial femoral condyle.

Answer: C – The McMurray test is performed by extending the fully flexed knee with the tibia internally rotated and applying varus stress, and by extending the fully flexed knee with the tibia externally rotated and applying valgus stress. Option A demonstrates the Steinmann test. Option B demonstrates the Apley test. Option D describes the Hughston’s plica test.

A physical therapist is evaluating a patient for abnormal sensations using a sharp-dull tool. Decreased sensation is noted in the medial aspect of the dorsum of the foot. Which of the following nerve roots is most likely damaged? A. T4 B. T10 C. L3

D. L5

Answer: D – An abnormal sensation in the medial aspect of the dorsum of the foot corresponds to L5 nerve root damage. Abnormal sensation in the nipple line suggests T4 damage. Altered sensations in the umbilicus and knee suggest damage of T10 and L3, respectively.

A physical therapist assesses the strength of the quadriceps femoris, which consists of the vastus lateralis, vastus intermedius, vastus medialis, and rectus femoris. The patient asks the patient to do which of the following during manual muscle testing? A. Sit with the knees over the side of the table, and extend one of the knee joints without rotating the thigh. B. Lie in prone position, and flex the

knee between 50-degrees and 70degrees with the thigh in slight lateral rotation. C. Sit the knee flexed to 90-degrees and then position the leg in lateral rotation of the tibia, medially rotate the tibia on femur. D. Stand on two feet, and plantarflex the bilateral ankles.

Answer: A – The quadriceps femoris is assessed by asking a patient to extend the knee. Option B assesses the biceps femoris. Option C evaluates the semimembranosus. Option D assesses the gastrocnemius. One complete gait cycle starts from the point at which the heel of one foot touches the ground and ends at the time when it touches the ground again. During the toe-off phase, the range of motion of the knee of a person with a normal gait is:

A. Fully extended B. Moving toward full extension C. Flexed at 35-degrees D. Flexed at 15-degrees

Answer: C – During the toe-off phase, the knee is flexed at 35degrees. The knee is fully extended on initial contact. During the loading phase, the knee is flexed at 15degrees. It moves toward full extension during the mid-stance and again fully extended during the terminal stance.

A male patient complains of cervical pain that radiates to the deltoid area and anterior aspect of the entire arm to the base of the thumb. Which of the following tests is performed by the physical therapist to assess for C5 radiculopathy? A. The patient pushes his arm away from the chest against resistance. B. The patient holds his extended fingers together against the therapist’s attempts to open the

fingers. C. The patient holds his shoulders in abduction against the downward force applied by the examiner. D. The patient lifts his arm against resistance by the examiner.

Answer: C – Weakness in shoulder abduction tests for C5 radiculopathy. Option A assesses for C7 radiculopathy. Option B assesses for C8 radiculopathy. Option D checks for C6 radiculopathy.

A physical therapist conducts sensory testing, mainly through light touch and pin sensation, on a patient with motor and sensory deficits. The therapist ensures to include which of the following areas in the test to determine whether or not patient has a motor incomplete injury? A. Medial aspect of the dorsum of the foot B. Anal sphincter C. Posterior aspect of the thigh

D. Groin

Answer: B – The anal sphincter is innervated by the S4-S5, representing the end of the spinal cord. The anal sphincter is a crucial part of the spinal cord injury examination because demonstration of voluntary anal sphincter contraction suggests an incomplete motor injury, regardless of any other finding. Abnormal sensation in the medial aspect of the dorsum of foot indicates L5 nerve root damage. Altered sensations in the posterior aspect of the calf and thigh suggest S2 nerve root damage. Abnormal

sensations in the groin indicate L1 damage.

A 41-year-old patient reports low back pain that radiates to the lower leg. Assessment of the patient includes positive straight leg raise test, asymmetric reflexes, and weakness. The patient is able to walk with difficulty. Admission is not required. Which of the following diagnostic tests is ordered for the patient? A. Magnetic resonance imaging B. CT scan

C. Myelography D. No diagnostic test is ordered

Answer: D – A diagnostic test is not ordered. The tests are ordered only if the patient is completely immobilized by pain and requires admission. If tests are considered, the magnetic resonance imaging is the modality of choice for patients with lumbar disk diseases.

A patient who complains of shoulder pain is assessed. It is noted that the pain is aggravated by movement and elevation of the arm above the shoulder level. The physical therapist tests the arm of the affected shoulder by internally rotating the patient’s arm and forcefully moves the arm through full range of forward flexion. The therapist performs which of the following tests? A. Neer’s test

B. Hawkin’s test C. Apley scratch test D. Yergason test

Answer: A – The Neer’s test is conducted to patients suspected of subacromial impingement. The test is positive if the patient complains of pain at 90-degrees of forward flexion. The Hawkin’s test is used in the diagnosis of supraspinatus tendon impingement. The Apley scratch test is used to detect a rotator cuff problem. The Yergason test is used if biceps tendon instability is suspected.

The ankle and foot work together during gait to provide the balance needed as the body moves over an uneven ground. The following indicate the normal averages and expected findings from mid-stance to toe-off except: A. The calcaneus inverts. B. The interphalangeal joints are in neutral position. C. The ankle moves from neutral to

10-degrees of dorsiflexion, progressing to about 20-degrees of plantar flexion on toe-off. D. The longitudinal arch lengthens.

Answer: D – At mid-stance, the longitudinal arch shortens. Inversion of the calcaneus and supination of the ankle occur to facilitate foot stability during propulsion at toe-off. The interphalangeal joints are neutral throughout the stance and swing phases. The ankle moves from neutral position to 10-degrees of plantar flexion after mid-stance. It moves to about 20-degrees of plantar flexion by the time of toe-off.

A female patient comes to the clinic complaining of acute pain in the left knee. The patient informs the physical therapist that she heard a popping sound in the knee as she was landing from jump. Anterior cruciate ligament injury is suspected. The physical therapist begins to perform which of the following as it is known as the most sensitive test for acute anterior cruciate ligament rupture? A. Pulling and pushing the proximal

part of the affected leg to test tibial rotation on the femur at the following: neutral, 30-degrees externally rotated, and 30-degrees internally rotated. B. Application of a valgus force to the knee, and at the same time, externally rotating the foot and passively straightening the knee. C. Application of pressure to the back of the knee as the thumb of the other hand attempts to displace the tibia forward from the femur.

D. Application of a valgus force to the knee as externally rotating the foot and passively straightening the knee.

Answer: C – The Lachman test, or the displacement of the tibia toward from the femur, is the most sensitive test for anterior cruciate ligament injury. The other tests are also performed to diagnose an ACL injury, but they are not the most sensitive test for this type of injury. Option A indicates the anterior drawer test, Option B is the McMurray test, and Option D is the pivot test.

A female patient reports radiation of cervical pain to the anterior left arm, radial side of the hand, thumb and to the index finger. Paresthesias in the left thumb and index finger are also noted. Which of the following tests is appropriately performed to the patient to assess for C6 radiculopathy? A. The patient pushes her arm away from the chest against resistance applied by the therapist.

B. The patient lifts her arm against resistance by the therapist. C. The patient holds her shoulders in abduction, against a downward force applied by the therapist. D. The patient holds her extended fingers together against the therapist’s attempts to open the fingers.

Answer: B – Weakness on elbow flexion and wrist extension as the patient lifts her arm against resistance by the therapist indicates C6 radiculopathy. Option A assesses for C7 radiculopathy. Option C assesses for C5 radiculopathy. Option D assesses for C8 radiculopathy.

A baseball player complains of shoulder pain when pitching. Anterior glenohumeral instability is suspected. On examination, a series of tests are conducted to aid the diagnosis. Which of the following tests, if positive, strongly suggests anterior glenohumeral instability? A. Apprehension test B. Sulcus sign C. Yergason test

D. Apley scratch test

Answer: A – A positive apprehension test is indicative of anterior glenohumeral instability. A positive sulcus sign suggests inferior glenohumeral instability. A positive Yergason test indicates the occurrence of biceps tendon instability. A positive Apley scratch test suggests a rotator cuff problem.

The patient sits with the elbow flexed at a right angle, with the forearm in supination. The physical therapist stands in front of the patient and places one hand over the flexor surface of the patient’s forearm. The other hand is applied to the humerus. The patient flexes the elbow against the therapist’s applied force. The therapist is assessing which of the following nerve roots? A. C5

B. C6 C. C7 D. C3

Answer: A – The therapist is assessing the motor function of the C5 nerve root. C6 is assessed by flexion of the forearm with brachioradialis as the segmentpointer muscle. C7 is assessed by extension of the forearm, with triceps as the segment-pointer muscle. C3 is evaluated by respirations.

A patient complaining of anterior shoulder pain is suspected of bicipital tendonitis. The physical therapist evaluating the patient conducts which of the following tests as a screening tool for diagnosing bicipital tendonitis? A. Yergason test B. Posterior drawer test C. Biceps load 2 test D. Anterior drawer test



Answer: A – The Yergason test is the screening tool for bicipital tendonitis. The posterior drawer test assesses the integrity of the posterior capsular structures. The biceps load 2 is specifically done to evaluate the SLAP lesions. The anterior drawer test detects the laxity of the anterior capsular mechanism.

A patient complaining of knee pain demonstrates joint line tenderness during assessment. Effusion, frank locking, and restricted motion of the affected knee are noted. History reveals that the patient is frequently required to perform frequent kneeling and squatting at work. A meniscus injury is considered. The following provocative maneuvers are performed except: A. Steinmann test

B. Apley test C. McMurray test D. Sag test

Answer: D – Sag test is conducted on patients suspected of posterior cruciate ligament injury. The Steinmann test, Apley test and McMurray test are the provocative maneuvers that create compression or shearing forces on the torn meniscus.

A patient with weakness and pain in the left shoulder is being evaluated. The symptoms are aggravated by overhead activities. Which of the following tests is conducted by the physical therapist to detect a tear involving the rotator cuff? A. Apprehension test B. Drop arm test C. Relocation test D. Hawkins’ test



Answer: B – The drop arm test is conducted to detect a rotator cuff tear. The therapist places the patient’s arm in maximum elevation in the scapular plane, and then lowers it slowly. A positive test is noted with dropping of the arm. Apprehension test and relocation test are done to detect anterior instability. The Hawkins’ test is used to detect impingement of the supraspinatus tendon.

A wrestler has significant tenderness and swelling on the inside of the left knee. The patient complains of knee instability when attempting to cut. A history of a blow to the outside of the left knee is assessed. Medial collateral ligament injury is suspected. The physical therapist conducts which of the following tests to evaluate the integrity of the medial collateral ligament? A. Varus stress test

B. Valgus stress test C. Posterior draw sign D. Anterior draw sign

Answer: B – A valgus stress test is conducted to evaluate the integrity of the medial collateral ligament. If any laxity is noted, the laxity should be compared to the opposite knee for grading. Options C and D are done to rule out associated injuries.

A physical therapist positions a patient in supine position to stabilize the scapula. The therapist brings the affected arm into an abducted and externally rotated position. The test causes the patient to guard arm movement, refusing further motion. The result is indicative of: A. Subacromial impingement B. Supraspinatus dysfunction C. Acromioclavicular joint disorder

D. Anterior glenohumeral instability

Answer: D – A positive apprehension test indicates anterior glenohumeral instability. Subacromial impingement is detected by the Neer’s test. Supraspinatus dysfunction is assessed by the supraspinatus isolation test or empty can test. The cross-arm test is used to assess for acromioclavicular joint dysfunction.

A physical therapist evaluates a female patient for the first time. The patient record reveals a diagnosis of trigeminal neuralgia and multiple sclerosis. To assess the cranial nerve involved in trigeminal neuralgia, the physical therapist performs which of the following tests? A. Weber test B. Bithermal caloric test C. Cotton-wisp test

D. Extra-ocular muscle test

Answer: C – A cotton-wisp test tests the trigeminal nerve or cranial nerve V. Options A and B test the eighth cranial nerve. Option D tests the fourth cranial nerve. Other techniques to evaluate for the fifth cranial nerve include lightly touching each side of the face and asking the patient to clench the jaw.

A physical therapist abducts the patient’s arm to 90-degrees and angles the arm 30-degrees forward at the shoulder joint. The patient is instructed to internally rotate the shoulder, as if pouring a can on the floor. The therapist screens for tears in which of the following muscles? A. Infraspinatus B. Subscapularis C. Serratus anterior

D. Supraspinatus

Answer: D – The empty can test is the screening tool for tears of the supraspinatus muscle. The infraspinatus is assessed by placing the patient’s arms at the sides, as the patient flexes both elbows to 90degrees as the therapist applies resistance against external rotation. The serratus anterior dysfunction is manifested by scapular winging. The subscapularis is assessed by the liftoff test.

A patient with a nonspecific shoulder pain with overhead activities is evaluated by the physical therapist. Stiffness, weakness, and popping sensations at the shoulder joint are reported. History includes a traction injury to the affected shoulder. SLAP lesion is suspected. Which of the following tests are conducted to aid the diagnosis of a SLAP lesion? I. Apley scratch test II. O’ Brien active compression test

III. Speed biceps tension test IV. Sulcus test A. I, II B. II, III C. III, IV D. I, III, IV

Answer: B – The O’Brien active compression and Speed biceps tension tests are conducted to detect labral damage. The Apley test is used to assess the rotator cuff. The sulcus test is conducted to evaluate inferior glumeral instability.

A physical therapist is evaluating a patient who complains of signs and symptoms indicative of lumbar radiculopathy. The therapist ensures to include which of the following maneuvers during the physical assessment as it is the most sensitive test for sciatic nerve irritation, inflammation or compression? A. Slump test B. Straight-leg raise test C. Contralateral straight-leg raise test

D. Bragard test

Answer: B – The straight-leg raise test is the most sensitive test for sciatic nerve compression. Sciatica may be caused by disk herniation, lumbosacral or cervical lesion, subluxation syndrome, tight hamstrings, or any condition that causes inflammation or irritation of the sciatic nerve. The SLR test is not specific. The slump test may add more specificity to SLR test. The contralateral straight-leg raise test is less sensitive for sciatica, but it is more specific for disk herniation. The Bragard test is a modification of

the SLR test.

A 50-year-old female complaining of cervical pain that radiates to the left lateral arm and forearm is suspected of cervical spondylosis. On physical examination, the pain in the left lateral arm and forearm is aggravated by lateral bending of the neck toward the side of pain and neck extension. The patient is positive for which of the following tests? A. Lhermitte sign B. Hoffman sign

C. Spurling’s test D. Adson’s test

Answer: C – The patient is determined to be positive for Spurling’s test. A Lhermitte sign is manifested by generalized shock sensation with neck extension. Hoffman sign is noted with reflex contraction of the thumb and index finger with nipping of the middle finger. Adson’s test is considered positive if the radial pulse is decreased or absent with abduction, extension, and external rotation of the upper extremity at the shoulder joint.

A patient with decreased range of motion of the cervical spine secondary to pain and muscle spasm is assessed for signs that may support the diagnosis of cervical spondylosis. The physical therapist notes that nipping of the middle finger elicited reflex contraction of the thumb and index fingers. Based on this finding, the patient is positive for which of the following signs? A. Hoffman sign

B. Lhermitte sign C. Spurling sign D. Yergason sign

Answer: A – The patient is positive for Hoffman sign. This finding is evidence of an upper motor neuron lesion. Lhermitte sign is manifested by generalized shock sensation with neck extension. Spurling sign is manifested by exacerbated radicular pain with neck extension and lateral bending of the neck toward the sign of lesion. Yergason sign is manifested by pain in the bicipital groove with supination and external rotation of the arm against resistance; this test is a screening tool for the diagnosis of bicipital tendonitis.



A patient complains of low back pain that radiates down to the leg. The patient reports that the pain is usually aggravated by bending or sitting and alleviated by standing. Which of the following tests is the most specific for disk herniation? A. Straight-leg raise test B. Contralateral-leg raise test C. Slump test D. Nachlas test



Answer: B – The contralateral-leg raise test is the most specific test for disk herniation. Straight-leg raise test is sensitive for sciatica, but it is not specific for disk herniation. The patient lies on her back with both legs straight. The examiner raises the affected leg upwards with the knee straight. If pain is reported down the back of the raised leg, the patient is positive for SLR test, suggesting nerve root irritation. If the unaffected leg is raised and pain is reported in the affected leg, it indicates disk herniation.



Carpal tunnel syndrome consists of signs and symptoms caused by the entrapment of the median nerve within the carpal tunnel. When evaluating a patient suspected of carpal tunnel syndrome, the physical therapist considers which of the following tests as the most specific for this condition? A. Square-wrist sign B. Phalen sign C. Carpal compression test

D. Palpatory diagnosis

Answer: C – The carpal compression test is the most sensitive and specific test for carpal tunnel syndrome. No good test is known to support its diagnosis, but research indicated that the carpal tunnel compression test has a sensitivity of about 89% and specificity of 96%. The square-wrist test has a sensitivity-specificity of 70%. The Phalen sign has a specificity of 0%; sensitivity is lower. Palpatory diagnosis has a specificity of 75%.

A female patient with neurological deficits is assessed by the physical therapist. When assessing the 11 th cranial nerve, also known as spinal accessory nerve, the therapist performs which of the following actions? A. The therapist asks the patient to push her tongue against a tongue depressor. B. The therapist is asked to identify a taste at the back of the tongue.

C. The therapist assesses sensation to the thoracic viscera. D. The therapist applies resistance to the trapezius muscle and asks the patient to shrug.

Answer: D – The therapist instructs the patient to shrug the shoulders as the therapist applies resistance. Option A tests the 12 th cranial nerve or hypoglossal nerve. Option B tests the 9th cranial nerve or glossopharyngeal nerve. Option C assesses the 10th cranial nerve or vagus nerve.

A female patient complains of dull aching pain in the legs after standing. A Trendelenburg’s test is performed to assess the competency of the valves of the superficial and deep veins. The test reveals rapid filling from the proximal end. This finding indicates: A. Normal, competent valves. B. Presence of varicosities. C. Incompetent communicating veins.

D. Claudication.

Answer: B – When veins fill from the proximal end, varicosities are present. When veins fill from the distal end, it indicates normal competent valves. If there is no rapid filling on standing and then there is a rapid filling after releasing the tourniquet, it indicates incompetent superficial veins.

A physical therapist conducts a thorough physical assessment on a patient suspected of sacral spine and pelvis dysfunction. The following tests are used to test the sacral spine and pelvis except: A. Approximation test B. Supine-to-sit test C. Gillet test D. Quadrant test

Answer: D – The quadrant test is used to assess for nerve-root irritation or facet pathology. Approximation test assesses for sacroiliac ligament sprains. Supineto-sit test is used to evaluate functional leg-length discrepancy secondary to a pelvic rotation. A Gillet test is used to assess for hypomobility of the sacroiliac joint on the examined side.

Foundations for Evaluation, Differential Diagnosis, and Prognosis

A damage involving the descending motor pathways and anywhere along this course causes a set of symptoms called the upper motor neuron syndrome. Which of the following assessment findings are consistent with an upper motor neuron lesion? I. Uncoordinated hyperreflex activities II. Weakness III. Absence of fasciculations

IV. Decreased muscle tone A. I, II B. I, III C. I, II, III D. I, II, IV

Answer: C – An upper motor neuron lesion is characterized by spastic paralysis, which is manifested by uncoordinated hyper reflex activities, weakness, absence of fasciculations, and increased muscle tone. Atrophy is usually minimal. A lower motor neuron lesion is characterized by flaccid paralysis, which is manifested by reduced movement, tone and reflex activity. The affected muscles are usually atrophied and flaccid.

A patient complains of pain in the left knee joint that is relieved by rest. The pain is aggravated after an activity. Crepitus, joint enlargement, and effusion are noted. The patient also reports that temperature changes aggravate the pain. Based on these findings, the patient is most likely diagnosed with: A. Rheumatoid arthritis B. Osteoarthritis C. Osteoporosis

D. Scoliosis

Answer: B – The patient is most likely diagnosed with osteoarthritis. Option A is manifested by joint pain that occurs in a symmetrical manner. Option C is manifested by loss of height and fractures. Option D is manifested by asymmetry of shoulders and lateral curvature and rotation on posteroanterior x-ray film.

A volleyball player complains of severe left shoulder pain. On initial assessment, the patient demonstrates decreased range of motion in the affected extremity. Physical examination findings include slight abduction and external rotation of the left arm. The left shoulder is squared off with absence of deltoid contour. The patient resists abduction and internal rotation and is not able to touch the unaffected shoulder. The patient is most likely diagnosed with:

A. Anterior shoulder dislocation B. Posterior shoulder dislocation C. Proximal humerus fracture D. Clavicular fracture

Answer: A – The patient is most likely diagnosed with anterior shoulder dislocation. In a posterior dislocation, the affected arm resists external rotation and abduction. Proximal humerus fracture is characterized by ecchymosis and edema of the affected shoulder with severe restricted range of motion. Clavicular fracture is manifested by the affected extremity that is adducted against the chest wall.

An elderly patient had a right knee surgery one week before the consult. Redness, swelling, and edema in the operated knee are noted. The knee is warm to touch. Infection is considered. Which of the following is asked by the physical therapist to determine the possible cause of the findings? A. “Are you sensitive to cold temperatures?” B. “Do you have a history of allergic

reactions?” C. “Have you had any rise in temperature?” D. The findings are expected; questioning is not necessary.

Answer: C – The signs and symptoms may indicate a case of infection. These findings are not expected one week after the surgery. A fever may further support a diagnosis of infection. A question focusing on changes due to cold temperatures is more appropriate for patients suspected of arthritis.

Gait assessment and balance evaluation is conducted on a patient who recently resumed full weight bearing after an injury. On assessment, a drop of the pelvis is noted toward the swing leg with visible movement of the hip toward the stance leg. The patient moves the trunk laterally over the stance-phase leg. The patient’s gait indicates: A. Quadriceps weakness B. Hip abductor weakness

C. Hip extensor weakness D. Hip flexion contracture

Answer: B – The patient’s gait indicates hip abductor weakness. The hip abductors stabilize the pelvis to limit its downward rotation in the frontal plane. In patients with hip abductor weakness, the pelvis drops toward the swing leg because the strength of the weight-bearing gluteus medius is insufficient to support the pelvis level. This is known as the uncompensated gluteus medius gait. The patient may move laterally over the weak hip to force the erector spinae and quadratus lumborum of the opposite side to

contract and lift the pelvis.

A patient is diagnosed with venous thrombosis, a condition that involves thrombus formation and inflammatory process. The following patients are at the risk to develop venous thrombosis except: A. A patient who had a hip surgery. B. A patient with venous stasis from varicose veins. C. A patient with a hypercoagulability disorder.

D. A patient with an arterial disorder.

Answer: D – A patient with an arterial disorder is suspected of peripheral artery disease. Options A, B and C are the risk factors for venous thrombosis. Injury to the venous wall from insertions, ulcerative colitis and pregnancy are the other risk factors for venous thrombosis.

A physical therapist understands that a patient with C8 tetraplegia will require assistance in some areas due to the limitations of the cervical spinal cord injury. To come up with a rehabilitation program that includes realistic and measurable goals, the therapist should consider that which of the following is most likely expected in patients with C8 tetraplegia? A. Absence of movement of upper and lower extremity muscles.

B. Good elbow flexion with absence of elbow flexion. C. Functional finger flexion. D. Absence of finger movement and elbow extension.

Answer: C – Functional finger flexion is noted in patients with C8 tetraplegia. The patient is able to perform activities that require hand grasp and release. Option A is noted in patients with C1-C4 tetraplegia. Option B is noted in patients with C5 tetraplegia. Option D is assessed in patients with C6 tetraplegia.

A physical therapist records the progress of a patient who is on a cardiac rehabilitation program. The therapist evaluates the progress based on the goals created before the start of program. Patient progress is documented in which of the following portions of the SOAP? A. Subjective B. Objective C. Assessment

D. Planning

Answer: C – The progress of a patient is documented on the assessment part of the SOAP. Proper documentation of the patient’s progress allows the therapist to evaluate the effectiveness or lack of effectiveness of implemented interventions. By assessing the outcomes, the therapist can make the necessary adjustments for maximal recovery.

A female patient complains of lower back pain that radiates down to the medial malleolus. On assessment, the patellar tendon reflex is decreased. The patient has difficulty walking on her toes. Sensation is decreased on the medial aspect of the tibia and medial malleolus. The patient is suspected of herniated nucleus pulposus. The root that is most likely affected is: A. L5

B. S1 C. L4 D. T10

Answer: C – The affected root is most likely L4. An L5 radiculopathy is manifested by pain radiating across the top of the foot, weak extensor hallucis longus, and decreased sensation along the lateral aspect of the leg. S1 radiculopathy is characterized by pain radiating to the lateral malleolus. A loss of sensation at the umbilicus suggests T10 radiculopathy.

Pain description is an essential part of the patient’s history. During pain evaluation, the location of pain, extension, onset, frequency, progression, intensity, and aggravating or relieving factors are noted. Which of the following pain measurements uses a number, such as 0 to 5 or 0 to 10, to reflect the decreasing or increasing degrees of pain? A. The Visual Analog Scale

B. Verbal Rating Scale C. Numerical Rating System D. Wong-Baker FACES Pain Rating Scale

Answer: C – The numerical rating system uses a number to reflect the intensity of the pain. In physical therapy, the two main pain measurements used are the Visual Analog Scale and the Numerical rating system. The NRS is easier to use than the VAS. The patient is usually asked, “If zero is equivalent to no pain, and 10 is the worst pain, how would you rate your pain?”

A 3-month-old child who tested positive for Ortolani and Barlow tests is referred to the physical therapist. The physical therapist reviews the patient’s record, expecting to note which of the following details of information in the patient history? A. The child is delivered in breech position. B. The child’s posterior shoulder got caught on its mother’s sacrum during

birth. C. The therapist is treating a male child. D. The child’s mother has a record of G3P3.

Answer: A – A breech position at birth increases the risk for developmental dysplasia of the hip. Other risk factors for DDH include female sex, genetic predisposition and being the first-born. Shoulder dystocia may be caused by difficult or prolonged labor, usually involving the posterior shoulder getting caught on the mother’s sacrum.

A physical therapist is treating an elderly patient after a prolonged admission in an acute care facility. The patient is maintained on pain medications and diuretics. Before starting an exercise session, the patient complains of fatigue, leg cramps and palpitations. The therapist suspects that these signs and symptoms are indicative of: A. Arterial claudication B. Hyperglycemia

C. Hypokalemia D. Deep vein thrombosis

Answer: C – The patient is most likely demonstrating the signs and symptoms of hypokalemia. Some diuretics cause the excretion of potassium. Hypokalemia is also manifested by irregular pulses.

A female gymnast complains of frequent burning pain above the right clavicle. The pain usually radiates to the right arm. The painful episodes are resolved in about two minutes without interventions. The patient is demonstrating signs of burners. Which of the following is the most common mechanism of injury of burners? A. Direct blow to the supraclavicular fossa.

B. Compression by a combination of hyperextension and ipsilateral lateral flexion. C. Percussive injury to the upper trunk. D. Traction injury to the brachial plexus.

Answer: D – Traction injury, in which the head and neck are forcefully moved away from the ipsilateral depressed shoulder, is the most common cause of burners. Options A and B are mechanisms of injury of burners, but they are not the most common. A direct blow to supraclavicular fossa causes percussive injury to the upper trunk.

A patient complains of pain and snapping in the medial region of the groin. On assessment, the symptoms are reproduced by active and passive flexion-extension of the hip. An audible snap is noted with the extension test. The patient is most likely diagnosed with: A. Iliotibial band syndrome B. Piriformis syndrome C. Iliopsoas bursitis

D. Meralgia paresthetica

Answer: C – The patient is most likely diagnosed with iliopsoas bursitis. Iliotibial band syndrome is characterized by pain in the lateral hip, thigh or knee and positive Ober’s test. Piriformis syndrome is characterized by pain on active external rotation and passive internal rotation of the hip. Meralgia paresthetica is manifested by abnormal distribution of lateral cutaneous nerve on sensory assessment.

A physical therapist is conducting a respiratory assessment on a patient before starting an exercise session. On auscultation, the therapist notes loud, high-pitched sounds over the sternum. This breath sound is identified as which of the following? A. Vesicular B. Bronchovesicular C. Bronchial D. Tracheal



Answer: C – Bronchial breath sound is characterized by loud, highpitched sound heard at the sternum. Option A is characterized by soft, low-pitched sound at the lung periphery. Option B is noted as a medium-pitched sound heard near the main stem bronchi. Option D is described as a loud and harsh sound heard on the trachea.

On gait assessment and balance evaluation, the patient keeps the knee extended at heel strike and through the stance phase. Knee flexion is reduced. The movement into full knee extension snaps the affected knee back. This finding indicates that the patient has: A. Ankle dorsiflexion weakness B. Quadriceps weakness C. Hip extensor weakness

D. Knee flexion contracture

Answer: B – The patient’s gait indicates quadriceps weakness. At heel strike, the quadriceps usually eccentrically contract to control limb loading and to prevent excessive flexion of the knee. The other options are not correct. Option A is manifested by food slap with reduced lateral ankle stability. Hip extensor weakness is manifested by slowed walking speed; the trunk is moved posteriorly by increased lumbar extension. Knee flexion contracture is manifested by lack of full knee extension in stance or short leg limp.



A male basketball player complains of acute pain in the left knee. On examination, hemarthrosis and inability of complete knee extension are noted. The patient informs the physical therapist that an audible pop was heard as he was changing direction during a game. Absence of endpoint is noted when the tibia is displaced forward from the femur. McMurray’s test and sag test are negative. The patient is most likely diagnosed with:

A. Anterior cruciate ligament injury B. Medial collateral knee ligament injury C. Posterior cruciate ligament injury D. Medial meniscus tear

Answer: A – The patient is most likely diagnosed with anterior cruciate ligament injury. A positive Lachman test occurs when an endpoint is not encountered. An audible pop is reported with anterior cruciate ligament injuries. A positive sag test indicates posterior cruciate ligament injury. A positive McMurray test is indicative of medial collateral injury or medial meniscus tear.

A female patient with impingement of the S1 nerve is being evaluated by a physical therapist. During the physical examination, which of the following is most likely noted by the therapist? A. Weakness in the gluteus medius. B. Weakness in the hamstring. C. Weakness in the quadriceps D. Weakness in the gastrocnemius.

Answer: D – Weakness in the large gastrocnemius muscle suggests S1 impingement. Options A and B are seen in patients with L5 impingement. Option C is assed in patients with L3 impingement.

After a thorough physical examination, a 5-month-old female is suspected of hip dislocation. The physical therapist reviewing the patient’s record expects to note which of the following findings as the most significant during the examination? A. A positive Ortolani test. B. A positive Galleazi sign. C. A positive Barlow test.

D. The child walks on the toes on the affected side.

Answer: B – A positive Galleazi sign is the most significant finding for a 5-month-old child. Ortolani and Barlow tests are usually performed in infants below 3 months. In infants between 3 and 6 months, the requirements of examination change. A positive Galleazi sign is manifested by one leg appearing shorter than the other when the child lies supine with the hips and knees flexed. In children older than 6 months or in children who can stand and walk, the examiner must note walking on the toes on the affected

side as indicative of hip dysplasia.

A patient with coronary artery disease and stable angina is referred for an exercise program. The physical therapist reviews the patient’s record, expecting to note which of the following risk factors for coronary artery disease? A. Female gender B. BMI of 28 C. Average blood pressure of 140/90 mm Hg

D. Diabetes mellitus

Answer: D – Diabetes mellitus is one of the many risk factors for coronary artery disease. The male gender, obesity or a BMI of 30 or greater, and hypertension with a systolic blood pressure greater than 160 mm Hg and diastolic blood pressure greater than 95 mm Hg are the other risk factors for CAD.

On gait assessment and balance evaluation, the patient demonstrates difficulty in advancing the trunk forward in terminal stance. The pelvis flexes forward to maintain vertical orientation of the trunk. Increased lumbar spine extension is observed. Based on these findings and on the gait pattern of the patient, which of the following conditions is most likely? A. Hip extensor weakness

B. Hip abductor weakness C. Hip pain D. Hip flexion contracture

Answer: D – Hip flexion contracture is most likely considered. In hip flexion contracture, abnormalities are obvious during the terminal stance, where maximal extension range is needed. The pelvis must flex forward to stabilize the trunk. Increased lumbar extension is the most common compensatory strategy used in patients with hip contractures less than 15-degrees. With hip extensor weakness, walking speed is slowed with increased lumbar extension or posterior trunk lean. Hip abductor weakness is manifested by pelvis

drop toward the swing leg and lateral movement of the hip toward the stance leg. Hip pain is characterized by trunk bending over the painful stance phase limb.

A patient with muscle weakness is suspected of a lesion in C8. The physical therapist evaluating the patient for the first time would expect incomplete paralysis of which of the following muscles? A. Supraspinatus B. Extensor carpi radialis C. Biceps D. Flexor carpi radialis

Answer: B – The C8 supplies the extensor carpi radialis. Myotomes are groups of muscles supplied by a single nerve root. A lesion in C8 is usually associated with incomplete paralysis or weakness of the ulnar deviators, thumb extensors, and thumb extensors. Weakness of the supraspinatus indicates a lesion in C5. Incomplete paralysis of the biceps suggests a lesion in C6. Paresis of the flexor carpi radialis is usually caused by a lesion in C7.

A 42-year-old male baseball player complains of shoulder pain that increases in severity with overhead activities. The pain is reported in the anterolateral aspect of the left shoulder radiating to, but not beyond, the elbow. The intensity of pain increases at night. A clicking sensation is also noted in the affected shoulder. On examination, increased external rotation and decreased internal rotation in the pitching arm are noted. As the physical therapist forcefully internally rotated a 90degree forwardly flexed arm, facial grimacing is observed. Drop arm test

is negative. The patient is most likely diagnosed with: A. Shoulder impingement syndrome B. Superior labral, anterior to posterior, or a SLAP lesion C. Bicipital tendonitis D. Rotator cuff tear

Answer: A – The patient demonstrates signs of shoulder impingement syndrome. Superior labral, anterior to posterior or SLAP lesions, are characterized by nonspecific shoulder pain and a positive O’Brien sign. Bicipital tendonitis is manifested by discrete pain and tenderness in the bicipital groove. Rotator cuff tear is manifested by a positive drop arm test.

Initial assessment reveals scapular winging. Which of the following are the probable diagnoses based on this finding? I. Impingement II. Serratus anterior dysfunction III. Trapezius dysfunction IV. Rotator cuff tear

A. I and II B. II and III C. III and IV D. I, II, and III

Answer: B – Serratus anterior and trapezius dysfunctions are the probable diagnoses based on this finding. Impingement is usually characterized by nighttime shoulder pain. Rotator cuff tear is characterized by infraspinatus or supraspinatus wasting.

On evaluation, the patient demonstrates extreme anterior rotation of the tibia on the sagittal plane during the mid and late stance phases. During stance, the patient’s left foot is observed to be in dorsiflexion. The heel rise is not evident. Which of the following is the most likely cause of the finding? A. Severe L4 or L5 radiculopathy B. S1 radiculopathy C. Peroneal nerve injury at the

fibular head D. Normal finding

Answer: B – The condition S1 radiculopathy is most likely considered. The patient demonstrates a gait pattern that suggests plantarflexor weakness. S1 radiculopathy, tibial neuropathy, peripheral neuropathies, myelodysplasia, and plexopathies cause plantarflexor weakness. Options A and C usually cause ankle dorsiflexion weakness.

A 30-year-old female gave birth to a male newborn with talipes equinovarus involving the right foot. On physical examination, which of the following assessment findings is most likely documented in the patient record? A. Everted, plantar-flexed, and abducted foot. B. Inverted, plantar-flexed, and adducted foot. C. Inverted, dorsi-flexed, and

adducted foot. D. Inverted, plantar-flexed, and abducted foot.

Answer: B – Talipes equinovarus is manifested by inverted, plantarflexed, and adducted foot. Option A describes talipes equinovalgus. Option C refers to talipes calcaneovarus. Option D describes talipes equinovalgus.

A physical therapist is assessing the respirations of a female patient with neurological deficits. The therapist notes irregular respiratory patterns with pauses at the end of inspiration and expiration. This finding indicates dysfunction in which of the following parts of the brain? A. Cerebral hemisphere B. Medulla C. Pons

D. Basal ganglia

Answer: C – The patient demonstrates apneustic breathing, which indicates a dysfunction in the middle or caudal pons. Damage in cerebral hemisphere is indicated by rhythmical breathing with periods of apnea known as Cheyne-Stokes respirations. Damage in the medulla is manifested by regular rapid and deep sustained respiration known as neurogenic hyperventilation. A dysfunction involving the basal ganglia is also indicated by CheyneStokes respirations.

Sensory testing is conducted on a patient who complains of back pain that radiates to the trochanter and groin. Abnormal sensation is reported with light touch in the groin area. The finding suggests nerve root damage involving: A. T10 B. L3 D. S2 D. L1



Answer: D – Abnormal sensations reported in the groin area suggests L1 nerve root damage. Abnormal sensation in the umbilicus suggests T10 nerve root damage. Altered sensations in the knee indicate L3 nerve root damage. Abnormal sensory test results obtained from the posterior aspect of the calf and thigh suggest S2 nerve root damage.

A patient with supraspinatus wasting tests positive for drop arm test. The result of the test is indicative of rotator cuff tear. The physical therapist expects an order for which of the following diagnostic tests as it is the criterion standard for diagnosing injuries to the rotator cuff? A. Ultrasonography B. Plain radiography C. Arthrography

D. Magnetic resonance imaging

Answer: D – Magnetic resonance imaging is the criterion standard for diagnosing rotator cuff injuries. Option A may be ordered, but the specificity of this test is dependent on the skills of the ultrasonographer. Arthrography is replaced by the MRI as the standard criterion for rotator cuff injury evaluation.

A patient with a history of head trauma is assessed using the Glasgow Coma Scale. The patient is able to localize painful stimulus, verbally respond in confused conversation, and open eyes in response to sound. The patient’s GCS score is: A. 10 B. 11 C. 12

D. 13

Answer: C – The patient has a Glasgow Coma Scale score of 12. Localization of painful stimulus = 5; confused conversation = 4; and eye opening in response to sound = 3. GCS score less than 8 indicates coma.

A physical therapist is assessing a patient who has had a stroke. A significant part of the evaluation consists of assessment of the ability to perform functional tasks. When assessing mobility, the therapist evaluates for the following except: A. Ability to ambulate on various surfaces. B. Ability to get on and off the floor. C. Sitting and standing balance.

D. Definition of required word assistance.

Answer: C – Sitting and standing balance is assessed when evaluating balance. Options A, B, and C are assessed when evaluating mobility. Ability to sit up and lie down, ability to transfer from one surface to another, and negotiation of stairs and curbs are also evaluated. Word assistance is defined on a scale from total assistance to independent.

A 50-year-old female is evaluated for chronic pain in the lower back, weakness, and numbness in the legs during walking. The symptoms are usually resolved by rest. Bending forward diminishes the pain. On physical examination, the patient is noted for wide-based gait, abnormal Romberg test, and thigh pain after 30 seconds of lumbar extension. The patient is positive for the stoop test and negative for straight leg raise test. Based on these assessment findings, the patient is most likely diagnosed with:

A. Lumbar spinal stenosis B. Lumbar disk herniation C. Arthritis of the spine D. Intermittent claudication due to arterial disease

Answer: A – The patient is most likely diagnosed with lumbar spinal stenosis. Lumbar stenosis is the narrowing of the vertebral canal and vertebral foramina. This condition typically demonstrates signs of neurogenic claudication, which is caused by increased metabolic demands of compressed nerve roots due to the stenosis. In contrast to vascular claudication, neurogenic claudication is not aggravated by biking and lumbar flexion and is not alleviated by standing. Diminished lumbar extension is the most

significant finding in lumbar spinal stenosis.

A patient comes to the clinic complaining of pain and clunking sound in the shoulder area with overhead motion. Based on the chief complaint, which of the following is the patient’s probable diagnosis? A. Impingement B. Labral disorder C. Rotator cuff tear D. Serratus anterior dysfunction

Answer: B – The probable diagnosis is labral disorder. Impingement is characterized by shoulder pain at night. Rotator cuff tear is manifested by wasting of the infraspinatus or supraspinatus. Serratus anterior dysfunction is manifested by scapular winging.

A patient was admitted to the facility with a diagnosis of spinal cord injury. The physical therapist reviews the patient’s record and notes that the injury is sustained by hyperextension. Which of the following describes hyperextension forces? A. Forcible forward bending B. Vertical force application C. Forcible backward bending

D. Excessive head rotation

Answer: C – Hyperextension forces involves forcible backward bending. Option A refers to hyperflexion. Option B refers to axial loading. Option D refers to excessive turning of the head.

A patient may recover after a stroke by natural neurologic recovery. This form of recovery allows improved motor control. Which of the following is a common pattern of motor recovery following a stroke? A. Upper extremity is more involved than the lower extremity at onset. B. Motor recovery in the upper extremity is greater than in the lower extremity. C. The prognosis for return of useful

hand function is favorable even when upper extremity paralysis is complete at onset. D. The severity of upper extremity weakness at onset is an insignificant predictor of eventual motor recovery.

Answer: A – The upper extremity is usually more involved than the lower extremity at onset. In spite of this, motor recovery in the UE is less than LE motor recovery during rehabilitation. Based on this finding, the severity of weakness in the UE at onset is a significant predictor of eventual motor recovery. The prognosis for return of useful hand function is unfavorable when the UE paralysis is complete.

A female gymnast diagnosed with peripheral nerve injury is referred for physical therapy. The acute nerve injury of the patient is thought to be caused by a stretch injury. Which of the following is the most likely mechanism of injury? A. Traction injury B. Compression syndrome injury C. Surgical incision D. Tourniquet paralysis



Answer: A – Traction, such as forcible depression of the shoulder, is a stretch injury. The nerves can be stretched from 10 to 20% before they are partially or completely damaged. Option B is a crush and percussion injury. Option C is a laceration injury. Option D is a mechanical injury. Nerve injury may also be caused by cold injuries, penetrating trauma, and high-velocity injuries.

A patient complains of pain over the acromioclavicular joint. Swelling, bruising, and a prominent clavicle are noted during initial assessment. The patient is diagnosed with acromioclavicular joint injury. Which of the following is the most common mechanism of acromioclavicular joint injury? A. Increased subacromial loading B. Outlet impingement C. A fall directly onto the acromion

with the arm in adduction D. A fall onto an outstretched arm

Answer: C – A fall directly onto the acromion with the arm in adduction is the most common mechanism of acromioclavicular joint injury. Option A is common in shoulder impingement. Option B is more commonly reported in patients with rotator cuff injuries. Option D may also cause acromioclavicular joint injury, but it is not the most common mechanism of injury.

Being able to distinguish neurogenic from vascular claudication is essential because each condition requires specific treatments. The absence of which of the following should suggest the diagnosis of neuroclaudication? I. Paralysis II. Paresthesia III. Pain IV. Pulselessness

V. Pallor A. I, IV, V B. II, III C. I, II, IV, V D. I, II, III, IV, V

Answer: A – The absence of paralysis, pulselessness, and pallor suggests the diagnosis of neurogenic claudication. The 5 Ps of vascular claudication include paralysis, paresthesias, pain, pulselessness, and pallor.

A patient with signs of nerve root damage is assessed by a physical therapist. A part of the physical examination is the manual muscle testing, which aims to grade muscle strength. The test reveals weakness with inversion and dorsiflexion of the left foot. This finding indicates damage of the nerve root: A. L4 B. L5 C. L3

D. L2

Answer: A – The finding of the manual muscle testing indicates damage of the L4 nerve root. L5 nerve root damage is manifested by muscle weakness with dorsiflexion of the great toe. L3 nerve root damage is characterized by muscle weakness with knee extension. L2 nerve root damage is assessed by weakness with hip flexion.

A 75-year-old female is referred for physical therapy after a stroke. The husband asks the therapist how long it will take for his wife to achieve recovery. Which of the following statements made by the therapist is the most appropriate? A. “The rate of recovery is different for each patient. However, in most cases, most recovery from stroke takes place in the first six months. Minor measurable improvement occurs after 12 months of onset.”

B. “Each patient has his or her own rate of recovery. However, in most cases, most recovery from stroke takes place in the first 12 months. Minor measurable improvement occurs after 24 months of onset.” C. “The rate of recovery after a stroke may be different for each patient. However, in most cases, most recovery from stroke takes place in the first three months. Minor measurable improvement occurs after six months of onset.”

D. “The recovery may take place over a long period of time.”

Answer: C – Most recovery from stroke occurs in the first three months, and only minor additional measurable improvement may occur after six months after the onset. For patients with significant partial return of voluntary movement, recovery may be achieved over longer period of time.

A patient referred to the clinic is diagnosed with Brown-Séquard syndrome, which was caused by a penetrating injury. On physical assessment, the physical therapist is most likely to note which of the following findings? A. Motor function in the upper extremities is significantly reduced. B. Motor function, vibration and proprioception are lost on the same side of the body as the lesion.

C. Bowel and bladder arreflexia. D. Sensations of light touch, temperature, and pain are lost on the same side of the body as the lesion.

Answer: B – The motor function, vibration, and proprioception are lost on the same side of the body as lesion in Brown-Séquard syndrome. Sensations of light touch, temperature, and pain are lost on the opposite side of the body as the lesion. Option A is manifested by central cord syndrome. Option C is assessed in patient in patients with conus medullaris syndrome.

A volleyball player seeks consult due to aching pain over the medial elbow. On physical examination, the patient demonstrates pain with resisted wrist flexion and resisted forearm pronation. Percussion of the ulnar nerve reveals a negative result. Based on the initial assessment findings, the patient demonstrates the clinical characteristics of which of the following conditions? A. Medial epicondylitis

B. Olecranon bursitis C. Ulnar collateral ligament injury D. Carpal tunnel syndrome

Answer: A – The patient is demonstrating signs and symptoms of medial epicondylitis. Injury of the ulnar collateral ligament is also characterized by medial elbow pain, but a negative result during percussion of the ulnar nerve rules out this diagnosis. Olecranon bursitis is manifested by posterior elbow swelling. Carpal tunnel syndrome is characterized by aching sensation in the ventral aspect of the wrist, radiating to the palm and fingers.



A patient with right knee pain is positive for the McMurray test, Steinmann test, and Apley test. An injury involving the lateral meniscus is suspected. The physical therapist expects the physician to order which of the following tests as it is considered as the criterion standard study for imaging meniscus changes? A. Magnetic resonance imaging B. Arthrography C. Plain radiography

D. Ultrasonography

Answer: A – The MRI is the criterion standard study for imaging meniscus injuries or changes. Arthrography used to be the standard imaging study of meniscal injuries. Plain radiography is ordered to rule out degenerative changes or fractures. Ultrasonography is more useful in evaluating medial collateral ligament.

A female athlete with a history of falling onto a hard floor is diagnosed with olecranon bursitis. Physical therapy is prescribed for increased recovery rate. The physical therapist reviewing the patient record expects to note which of the following assessment findings documented during the initial physical examination? A. Maximal tenderness over the lateral epicondyle

B. Pain with resisted wrist flexion C. Swelling of posterior elbow D. Elbow pain when making a clenched fist

Answer: C – Swelling of posterior elbow is a classic sign of olecranon bursitis. Option A is assessed in patients with lateral epicondylitis. Option B is noted with medial epicondylitis. Option D is assessed in ulnar collateral ligament.

A female patient with muscle weakness in the supraspinatus, deltoid, and infraspinatus has decreased sensations over the deltoid area and the anterior aspect of the entire arm to the base of the thumb. On assessment, which of the following reflexes is most likely affected? A. None B. Triceps C. Biceps

D. Knee jerk

Answer: C – The biceps reflex is most likely affected. The patient demonstrates a possible lesion in C5. A lesion in this level usually affects the biceps and brachioradialis reflexes. Lesions in C1, C2, C3, C4, L1, and L2 do not usually causes changes in reflexes. The triceps reflex is affected by lesions in C7 and C8. Lesions in L4 and L5 alter the knee jerk.

A female was admitted to the facility due to a stroke. The patient is referred for physical therapy. The physical therapist considers the recovery rate of the affected extremities to create a treatment program that appropriately addresses the patient’s needs. Assessment of which of the following within 72 hours after stroke are useful when predicting upper limb recovery? I. Finger extension

II. Elbow flexion III. Shoulder abduction IV. Shoulder flexion A. I, II B. I, III C. I, IV D. I, II, III, IV

Answer: B – Assessment of the finger extension and shoulder abduction within 72 hours of stroke may be used to predict the recovery of the upper limbs. A study by Nijland et al revealed that if, by the second day following stroke, a patient begins to demonstrate voluntary extension of the fingers and abduction of the affected shoulder, there is a 0.98 probability that dexterity is regained by 6 months.

A patient complains of pain and cramping in the lower left leg. The pain usually occurs with walking and is relieved by rest. Neurogenic claudication and vascular claudication are considered. To distinguish one from the other, a detailed history is obtained. Which of the following details of information suggests claudication due to inadequate blood flow to the muscles? A. The leg pain is aggravated by

standing erect and downhill ambulation. B. The leg pain is not exacerbated with lumbar flexion. C. The leg pain radiates upward. D. The leg pain is relieved by standing.

Answer: C – In vascular claudication, the pain usually radiates upward. The pain usually radiates downward with neurogenic claudication. Vascular claudication is caused by insufficient supply of oxygen to the leg muscles, secondary to narrowing of the arteries. In contrast to neurogenic claudication, leg pain is aggravated by biking, uphill ambulation and lumbar flexion. It is relieved by standing.

A patient with left-brain damage is referred to the clinic for maximal motor and sensory recovery. The physical therapist assesses the patient, expecting to note the following findings except: A. Paralyzed right side B. Impaired speech C. Slow performance D. Impaired judgment

Answer: D – Impaired judgment is usually seen in patients with rightbrain damage. Left-brain damage is manifested by a paralyzed right side, impaired speech, and slow performance. Brain damage involving the right brain is manifested by impaired judgment and paralyzed left side.

A physical therapist is reviewing the chart of a patient who is recently referred to the outpatient clinic. It is documented that the patient has had an air plethysmography. The therapist is correct to conclude that the patient was most likely suspected of: A. Chronic venous insufficiency B. Chronic obstructive pulmonary disease C. Muscular wasting

D. Vestibular dysfunction

Answer: A – Air plethymosgraphy is a diagnostic test that quantitatively evaluates volume changes of the entire lower leg from knee to ankle. This test is ordered to patients with chronic venous insufficiency. Pulmonary plethymosgraphy measures the volume in the lungs when the muscles of respiration are relaxed. Muscular wasting may be assessed by electromyography. Vestibular dysfunction is assessed by bithermal caloric testing.

A white female is referred to the clinic for the treatment of carpal tunnel syndrome. The physical therapist reviews the patient record to create a treatment program that is most appropriate for the patient. The therapist expects to find which of the following assessment findings on the record? A. Pain with resisted wrist flexion B. Pain with resisted wrist extension C. Pain during valgus stress with the

elbow in 25-degrees of flexion D. Pain in the ventral aspect of wrist that radiates to the ventral forearm

Answer: D – Carpal tunnel syndrome is characterized by pain in the ventral aspect of the wrist that radiates to the ventral forearm. Option A is a finding in medial epicondylitis. Option B is characterized by lateral epicondylitis. Option C indicates ulnar collateral ligament injury.

A female patient with axonotmesis or Grade 2 nerve injury is evaluated before the start of the physical therapy program. The nerve injury occurred one week before the referral. The following signs and symptoms are most likely noted by the therapist during the physical examination except: A. Muscle wasting B. Complete sensory functions are lost

C. Complete motor functions are lost D. Absence of pain in the affected extremity

Answer: D – Absence or pain, or anesthesia, is commonly seen in patients with neurotmesis. In contrast to axonotmesis, neurotmesis involves destruction of the structure of nerve. Axonotmesis is manifested by pain, muscle wasting, and lost motor, sensory, and sympathetic functions.

A physical therapist is assessing an elderly female who recently had a total hip replacement surgery. When evaluating for the patient’s cardiopulmonary status, the therapist understands that which of the following changes are related to aging? A. Increased heart rate with exertion B. Increased maximal exercise capacity C. Decreased resting blood pressure

D. Increased work of breathing

Answer: D – Increased work of breathing is commonly assessed in the elderly. This is due to increased thoracic kyphosis. With aging, the heart rate with exertion and maximal exercise capacity decrease. The resting blood pressure increases with aging.

A female patient complains of decreased range of motion, spasm, and pain in the cervical spine due to trauma. Magnetic resonance imaging reveals degeneration of the seventh cervical spine. Which of the following is most likely included in the assessment findings? A. Diminished biceps reflex B. Muscle grade of 3 for the supraspinatus muscle C. Paresthesias in the index, long and

ring fingers D. Decreased sensations in the medial arm and forearm to long fingers

Answer: D – Decreased sensations in the medial arm and forearm is most likely assessed. In the cervical spine, a change occurring in the seventh cervical vertebra, where the C8 spinal nerve exits, causes symptoms related to C8 compression. A diminished biceps reflex occurs with C5 and C6 nerve root compressions. Weakness in the supraspinatus suggests compression of the C5 nerve compression. Paresthesias in the index, long and ring fingers indicate C7 nerve root compression.

Physical assessment of the musculoskeletal system is conducted through inspection, palpation, muscle mass measurement, and range of motion. It also follows a general sequence. A physical therapist conducting a musculoskeletal examination begins by: A. Inspection and palpation of bones B. Assessment of gait and posture C. Measurement of the length and circumference of the extremities

D. Assessment of muscle mass

Answer: B – A musculoskeletal examination begins by assessing the patient’s gait and posture. The general sequence is: assessment of gait and posture, inspection and palpation of bones, measurement of the length and circumference of the extremities, assessment of muscle mass, and assessment of joints for swelling, redness, pain, crepitus, range of motion, and warmth.

The lateral lumbar spine radiographs demonstrate significant changes with narrowing of the interspace and osteopathic canal. The patient is diagnosed with lumbar spinal stenosis. Which of the following findings during physical assessment is most consistent with the diagnosis? A. Onset of symptoms at rest. B. The low back pain is aggravated by lumbar flexion.

C. Bilateral leg weakness. D. Thigh pain after 30 seconds of lumbar extension.

Answer: D – Thigh pain is usually reported after 30 seconds of lumbar extension. A patient with lumbar spinal stenosis usually reports onset of symptoms when walking. The pain is relieved by rest. Lumbar flexion relieves the pain. Bilateral leg weakness is rare in patients with this condition.

A runner who sustained Grade III medial meniscal injury is considering having a meniscal repair. The patient understands that conservative treatment may delay her training. When asked how long would it take her to return to normal activity after the surgery, the physical therapist is correct to inform her that: A. She needs to avoid weight bearing 2 to 4 weeks after the surgery. B. Meniscal repair may interfere

with concomitant rehabilitation of the anterior cruciate ligament rehabilitation. C. Full weight bearing with the knee braced and in locked in full extension for the first 6 weeks after the surgery will be implemented. D. Avoidance of full motion when the knee is not bearing weight for the first 6 weeks after the surgery will be implemented.

Answer: C – After a meniscal repair, the patient has to have her knee braced and in locked in full extension for the first 6 weeks. Full motion of the knee is encouraged when it is not bearing weight. Arthroscopic repair may have a shorter recovery time. Normal activity after this lesser invasive surgery may be resumed two weeks after the procedure.

A patient with neuropraxia had a surgery that involved direct reconnection of the divided peripheral nerve. The physical therapist is aware that the signs and symptoms of motor and sensory recovery follow a particular sequence and duration of time. The therapist expects: A. Motor recovery of the distal muscles before motor recovery of the proximal muscles.

B. Motor recovery before sensory recovery. C. Return of voluntary control before muscles response to faradic stimulation. D. Deep cutaneous sensibility as the first sign of sensory recovery.

Answer: D – Recovery of deep cutaneous sensibility, or pain caused by deep pressure, is the first sign of sensory recovery. Sensory recovery occurs before motor recovery. The proximal muscles recover first before the distal muscles recover. Faradic stimulation, or stimulation applied to nerves at muscle motor points, elicit muscle response before the return of voluntary control.

A physical therapist is evaluating an elderly female for the first time. History of the patient includes a recent left hip fracture. On physical assessment, the therapist notes a pressure ulcer on the patient’s left heel. Further evaluation reveals a shallow open ulcer that appears like an open serum-filled blister with a red wound bed. Partial thickness loss of the dermis is noted. Based on these findings, the patient’s pressure ulcer is classified as:

A. Stage IV B. Stage III C. Stage II D. Stage I

Answer: C – The patient has a stage II pressure ulcer. Stage I is manifested by persistent and nonblanchable edema. Stage III involves full thickness skin loss with visible subcutaneous fat. Stage IV is manifested by full thickness tissue loss with exposed bone, tendon, or muscle.

A male patient with pain in the cervical spine reports radiation of pain to the medial arm and forearm, and to the long, ring and little finger. During assessment for cervical radiculopathy, the physical therapist conducts which of the following tests as it is the best test to verify the diagnosis of cervical radiculopathy? A. Lhermitte sign B. Manual cervical traction C. Foraminal compression test

D. Hoffman sign

Answer: C – The foraminal compression test, also known as Spurling test, is the best test for verifying the diagnosis of cervical radiculopathy. The Spurling test involves positioning the patient with the neck extended and the head rotated. A compressive force toward down the head is applied. If the pain extends to the extremity on the same side to which the head is rotated, the patient is said to be positive for Spurling test.

A patient who complains of leg pain with walking is being evaluated. The pain is often relieved by rest and bending forward. The physical therapist suspects lumbar stenosis. Which of the following tests is most likely initially ordered for the patient? A. Magnetic resonance imaging B. Plain radiography C. CT scan

D. Nuclear imaging

Answer: B – Plain radiography is the initial test ordered for a patient suspected of lumbar spinal stenosis. The MRI is the diagnostic test modality of choice. The CT scan can demonstrate the lateral access, neuroforaminal visualization, and central canal. Nuclear imaging is ordered if the physician suspects an existing disease related to the bones of the vertebra.

A patient with low back pain comes to the clinic for consult. The physical therapist understands that low back pain is caused by many diseases, some of which are not directly caused by spinal problems. To make an accurate diagnosis, which of the following is the priority action during clinical evaluation? A. Analysis of the chief complaint. B. Evaluation of the cervical and lumbar spine.

C. Identification of risk factors. D. Obtaining a history of activity intolerance.

Answer: A – The priority action is to make an analysis of the chief complaint. The therapist should ask for the dominating concern, followed by the frequency of the pain. Aggravating and alleviating factors are also obtained. The other options are essential during evaluation, but it is a priority to analyze the nature of the chief complaint.

A 60-year-old female complains of low back pain. The history reveals a recent fracture involving the forearm. The patient is also noted of loss of height and progressive curvature of the spine. Osteoporosis is being considered. Which of the following tests most accurately measures the bone density of the affected spine? A. X-rays B. Ultrasound C. Dual-energy x-ray absorptiometry

D. Quantitative tomography

computed

Answer: C – Dual-energy x-ray absorptiometry or DEXA is the gold standard for measuring bone density. The National Osteoporosis Foundation recommends BMD screening to women older than 65 years and to men older than 70 years, regardless of fracture risk.

A physical therapist is reviewing the history of a female patient who is recently referred for total rehabilitation program. The list reveals that the patient has Cushing’s syndrome and is maintained on drug treatment. Based on this detail of information, the patient is at risk for which of the following conditions? A. Diabetes mellitus B. Osteoporosis C. Hypotension

D. Weight loss

Answer: B – The patient is at risk for osteoporosis. A person diagnosed with Cushing’s disease is maintained on steroids throughout life. Steroid use inhibits the absorption of calcium in the gastrointestinal tract. Steroids do not cause diabetes mellitus. They elevate blood glucose, making it more difficult for patients with diabetes to control their blood sugar levels. Steroids may also cause hypertension, weight gain and redistribution of fat in the abdomen, face and between the scapulas.

A 50-year-old patient demonstrates decreased range of motion of the cervical spine secondary to pain. The biceps and brachioradialis reflexes are diminished. The patient is suspected of cervical spondylosis. Which of the following is the routine test ordered for patients with suspected cervical spondylosis? A. Plain radiography B. CT scan C. Magnetic resonance imaging

D. Myelography with CT scanning

Answer: A – Plain radiography is the routine test ordered for patients suspected of cervical spondylosis. CT scan is recommended for patients with acute fractures, especially those affecting the C6 and C7. MRI is the method of choice for imaging disc herniations. Myelography with CT scanning is the imaging of choice for spinal and foraminal stenosis.

A female patient reports brief and paroxysmal facial pain, which is described as severe in intensity and stabbing in quality. The pain typically starts on one side of the cheek and then radiates to the jaw, top lip, teeth and gums, and to the side of the nose. The pain is usually triggered by vibration, light touch, and face washing. The patient is most likely diagnosed with: A. Multiple sclerosis

B. Temporomandibular joint disorder C. Trigeminal neuralgia D. Migraine

Answer: C – The patient is most likely diagnosed with trigeminal neuralgia. Multiple sclerosis is manifested by significant fatigue, sensory deficits and optic neuritis. Temporomandibular joint disorder is manifested by deep ache associated with chewing, limited jaw opening, and popping or snapping sounds associated with the pain. Migraine occurs from four to 72 hours.

Osteoporosis is a metabolic bone disorder characterized by reduction of bone mass, increased fragility of bones and increased risk of fractures. When assessing a female patient, which of the following details of information increases her risk for osteoporosis? A. African American descent B. Potassium deficiency C. High fat intake

D. Cigarette smoking

Answer: D - Cigarette smoking has been considered as one of the risk factors for osteoporosis. Smoking decreases the blood supply to the bones, and nicotine inhibits the production of osteoblasts. European Americans and Asians are more at risk for osteoporosis. Calcium deficiency and high alcohol intake may also increase the risk.

A patient with minimal pain and hemarthrosis in the left knee tests positive for posterior tibial sag. Posterior cruciate ligament injury is suspected. Which of the following details of information provided by the patient during the history taking supports the diagnosis? A. A force applied to the anterior aspect of the patient’s proximal tibia when the knee is flexed. B. An outwardly-directed force

applied to a partially flexed knee from the inside. C. An inwardly-directed force applied to the knee from the outside. D. An audible pop in the knee after changing direction.

Answer: A – The mechanism of injury of posterior cruciate ligament injury usually involves application of force applied to the anterior aspect of the proximal tibia when the knee is flexed. Option B is the mechanism of injury for medial collateral ligament injury; option C for lateral collateral ligament injury, and Option D for anterior cruciate ligament injury.

An 11-year-old girl has an awkward walking pattern. When the adolescent stands, the knees touch but the ankles are apart. The patient complains of discomfort on the inside of the knees. Gait pattern is circumduction. Based on these assessment findings, the patient demonstrates signs of: A. Genu valgum, physiological B. Genu varum, physiological C. Genu valgum, pathological

D. Genu varum, pathological

Answer: C – The patient demonstrates genu valgum, pathological. Genu valgum, also known as knock knees, is usually seen by age 3. If the condition is observed after the age of 7, a night brace may be ordered. Genum valgus, also known as bowlegs, is usually seen in children younger than 18 months.

A patient with cervical pain that radiates to the deltoid area and anterior aspect of the entire arm to the base of the thumb is being evaluated by the physical therapist. Physical examination reveals a positive result for foraminal compression test or Spurling test. Cervical radiculopathy is suspected. Which of the following is the diagnostic method of choice for the evaluation of cervical radiculopathy? A. Magnetic resonance imaging

B. Plain radiography D. CT scan E. Myelography with CT scanning

Answer: A – The MRI is the diagnostic method of choice for the evaluation of cervical radiculopathy. Plain radiography is ordered for patients suspected of cervical disc disease as the cause of the radiculopathy. CT scans are most useful when evaluating acute cervical fractures. CT scanning is ordered for the diagnosis of cervical disc herniation.

A 35-year-old male is suspected of spinal cord injury after a motorcycle accident. Diagnostic tests indicate complete spinal cord injury involving T5-T6. The patient asks if he can still have an erection. Which of the following is informed to the patient? A. An erection is not possible with direct stimulation of the genitalia and cognitive stimulation. B. An erection is possible with direct stimulation of the genitalia and

cognitive stimulation. C. An erection is only possible with direct stimulation of the genitalia. D. An erection may occur with mental images, but not with direct stimulation of the genitalia.

Answer: C – An injury involving the T5-T6 segment preserves reflexogenic capacity, where erections are achieved with direct stimulation of the genitalia. Psychogenic erections, or erections stimulated by cognitive stimulations, are not possible. Males with complete SCI above T6 cannot transmit psychogenic input from the brain to the sacral levels.

A football player sustained an injury due to a valgus force applied to the knee when it was partially flexed during a game. A triad of injury sustained from this mechanism of injury involves the following structures in the knee except: A. medial collateral ligament B. medial meniscus C. patella D. anterior cruciate ligament



Answer: C – Patella is not a part of the triad. When a valgus force is applied, injuries to the medial collateral ligament, medial meniscus, and ACL commonly occur in athletes. An injury to the MCL is manifested by pain and stiffness in medial knee.

A female patient complains of acute onset of lower back pain that radiates to the lateral malleolus of the left extremity. On assessment, the ankle reflex is decreased. Ambulating on her toes is not tolerated due to pain. Sensation is decreased over the lateral malleolus, lateral foot and little toe. The patient is most likely considered for: A. L5 radiculopathy B. T12 radiculopathy

C. S1 radiculopathy D. L4 radiculopathy

Answer: C – The patient is most likely considered for S1 radiculopathy. L5 radiculopathy is manifested by lower back pain radiating across the top of the foot and decreased sensation along the lateral aspect of the leg. T12 radiculopathy is manifested by loss of sensation in the inguinal crease. L4 radiculopathy is characterized by low back pain that radiates to the medial malleolus.

A patient demonstrates signs of femoral nerve dysfunction. The patient reports a feeling of the knee giving away and weakness of the knee when going down the stairs. On assessment, decreased sensation over the thigh and knee is noted. Which of the following is most likely included in the patient’s record? A. A recent surgery in lithotomy position. B. A recent lower abdominal

incision. C. A recent meniscus repair. D. A recent appendectomy.

Answer: A – A common cause of femoral nerve dysfunction is lying on the back with the thighs and legs flexed, also known as the lithotomy position, during surgery. A recent lower abdominal incision, such as appendectomy incision, may cause entrapment of the ilioinguinal and iliohypogastric nerves. A recent meniscus repair may compress the saphenous nerve.

A physical therapist understands that when evaluating patients with T10L1 spinal cord injuries, the following assessment findings are most likely noted except: A. Normal shoulder movement B. Partial paralysis of the lower body and legs C. Partial use of arms, wrists and fingers D. Normal respiratory system



Answer: C – In patients with T10-L1 spinal cord injuries, full use of arms, wrists, and fingers are noted. The function of the arms, wrists, and elbows are affected in spinal cord injuries that involve the cervical spine.

A patient referred for physical therapy has a history of neck pain that radiates to the posterolateral arm and shoulder. Muscle weakness, reflex changes and paresthesias in the middle and index fingers are recorded in the patient record. On evaluation, the patient demonstrates muscle weakness of the wrist flexors, finger extensors, and triceps. Triceps reflex is decreased. The patient is most likely diagnosed with: A. C7 radiculopathy

B. C8 radiculopathy C. C5 radiculopathy D. C6 radiculopathy

Answer: A – The patient has C7 radiculopathy. This nerve root disease is the most common type of cervical radiculopathy. C8 radiculopathy is manifested by pain in the ulnar forearm and hand and numbness in the medial forearm and small finger. C5 radiculopathy is characterized by pain in the shoulder and weakness of the deltoid and biceps. C6 radiculopathy is manifested by pain in the shoulder, upper arm and lateral forearm and weakness in the biceps and wrist extensors.



A patient who is diagnosed with Guillain-Barré Syndrome is referred to the clinic for further treatment. The physical therapist understands that as a lower motor neuron disorder, the patient is most likely to demonstrate which of the following clinical manifestations? A. Clonus B. Spastic paralysis C. Positive Babinski reflex

D. Muscular fasciculations

Answer: D – Muscular fasciculations are noted in patients with lower motor neuron lesions. Other manifestations include flaccid paralysis, muscular atrophy, and hyporeflexia. Options A, B, and C are manifestations of an upper motor neuron lesion.

Interventions

A total rehabilitation program consists of two basic elements: therapeutic modalities and therapeutic exercise. For a therapeutic exercise to be effective, specific parameters must be followed sequentially. These parameters in their proper sequence are: I. Flexibility and range of motion II. Strength and muscle endurance III. Proprioception, coordination, and agility

A. I, II, III B. I, III, II C. II, I, III D. III, I, II

Answer: A – The correct sequence is: flexibility and range of motion, strength and muscle endurance, and proprioception, coordination, and agility. The sequence must be followed for a therapeutic exercise program to be effective. It is essential that each parameter is restored to at least its pre-injury levels before proceeding.

Non-steroidal anti-inflammatory drugs are the most frequently used anti-inflammatory drugs for the treatment of athletic and non-athletic musculoskeletal injuries. Which of the following NSAIDs is the most recommended drug for a patient who has a history of peptic ulcer disease? A. Ibuprofen B. Celecoxib C. Naproxen

D. Sulindac

Answer: B – Celecoxib is the most recommended NSAID for patients with peptic ulcer disease. NSAIDs reduce pain and manage inflammation by inhibiting the enzyme cyclooxygenase, an enzyme essential for prostaglandin synthesis. Celecoxib is a COX-2 inhibitor, which selectively inhibits COX-2. The other options include drugs that inhibit COX-1 and COX-2. COX-1 is essential in producing a protective lining along the inner stomach. Sparing the COX-1, COX-2 inhibitors reduce the risk for gastric

irritation.

A patient who was admitted due to myocardial infarction is referred for cardiac rehabilitation. This program is divided into four stages: acute phase, convalescent phase, training phase, and maintenance. Which of the following is included in the treatment plan during the first phase of cardiac rehabilitation? A. Emphasis on exercise intensity that is limited to a safe target heart rate.

B. Passive range of motion exercises. C. Achievement of 85% of the maximum heart rate. D. Symptom-limited full-level exercise tolerance.

Answer: B – Passive range of motion exercises are included in the first phase of rehabilitation. Option A is initiated on the second phase. Option C is initiated on the third stage. Option D is included to the fourth stage of the therapy.

A football player complains of frequent burning pain in the supraclavicular area that radiates down the left arm in a circumferential manner. Numbness and paresthesias in the affected extremity are also reported. The burning pain is usually resolved in two minutes without pain relief measures. Which of the following is a priority intervention to resolve burners? A. Improve aerobic capacity

B. Maintenance of chest-out posture C. Corticosteroid administration D. Heat therapy

Answer: B – Maintenance of chestout posture must be encouraged to resolve the burning sensation. This posture opens the foramina maximally, reducing the effect of the weight of the head on the nerve roots. This strategy decreases the pressure exerted on the brachial plexus by the scalene muscle. Other interventions include flexibility and strengthening exercises and protective gear-use. Antiinflammatory medications such as ibuprofen may be prescribed. Ice is applied to the area a few days after

the injury to resolve the swelling.

Chest physiotherapy uses percussion and vibration over the thorax to loosen the respiratory secretions in the affected segments of the lung. Which of the following is the best time to perform chest physiotherapy? A. Before sleeping. B. 2 to 3 hours before meals. C. 2 to 3 hours after meals. D. After a morning exercise.

Answer: C – The best time to perform chest physiotherapy is 2 to 3 hours after meals. CPT is also recommended 1 hour before meals and in the morning after arising. If the therapy causes pain, the therapist must stop.

Manipulative care is included in the comprehensive program for a female patient with a musculoskeletal problem involving the neck. Before the start of the therapy, the patient should be informed that which of the following is most likely to occur as a side effect of manipulation? A. Increased or early menses B. Weakness C. Increased sweating

D. Increased neck discomfort

Answer: D – A transient increase in discomfort between six and 72 hours is the most commonly reported side effect of manipulation. The other less common effects include temporary autonomic effects, such as early or increased menses, weakness, increased sweating, increased temperature, and skin redness.

A patient who sustained trauma to the chest complains of pain that increases with inspiration. Shallow respirations are noted. A rib fracture is considered. Which of the following interventions is the most appropriate for the patient? A. Instruct the patient to self-splint with hands and arms. B. Oxygen administration. C. Mechanical ventilation with PEEP.

D. Encourage coughing.

Answer: A – The patient is instructed to splint with hands and arms. Rib fractures usually heal spontaneously. Oxygen administration is more appropriate for patients with flail chest and pulmonary contusion. Mechanical ventilation with PEEP may be given to a patient with pulmonary contusion. Coughing is recommended to patient with flail chest.

A physical therapist is evaluating a patient who has C6 spinal cord injury. The physical therapist understands that the patient may perform which of the following activities independently: A. Clearing of respiratory secretions and coughing B. Washing C. Bowel and bladder management D. Brushing the teeth



Answer: D – The patient has the ability to brush teeth using palm straps. A patient with C6 spinal cord injury has a good wrist extension and elbow flexion. The patient may shave and brush hair with the use of assistive devices. A patient with C6 injury has complete paralysis of the body and lower extremities. Finger movement, elbow extension, and wrist flexion are not noted.

Traditional non-operative treatment for clubfoot deformities usually begins with application of splintage two to three days after birth. Which of the following is first step to correct the deformity? A. Equinus B. Forefoot supination C. Pronation D. Forefoot adduction

Answer: D – Forefoot adduction is the first step of correction. The next step is forefoot supination. Equinus is the last step of the treatment. Attempting to correct the equinus first may break the foot, causing a rockerbottom foot.

A physical therapist is treating a patient diagnosed with chronic bronchitis, a chronic pulmonary disease that secretes large amounts of respiratory secretions. When suctioning the patient, the following actions by the physical therapist ensure safety and maximal benefit of the therapy except: A. Use of sterile technique. B. Suctioning intermittently for 10 seconds.

C. Lubricating the catheter with sterile water before suctioning. D. Application of suction when inserting the catheter.

Answer: D – Suction should not be applied when inserting the catheter. This action can cause injuries to the mucus membranes. A sterile technique is used. Suctioning should be done in 10 seconds. The catheter must be lubricated with sterile water.

A physical therapist is planning a treatment program for a patient who is recently diagnosed with lumbar spinal stenosis. The program would consist of conservative treatment modalities, including exercise and activity. Which of the following is least likely included in the exercise program? A. Lumbar extension exercises B. Spinal flexion exercises C. Abdominal muscle strengthening

D. Inclined treadmill testing

Answer: A – Lumbar extension exercises are avoided. This type of exercise encourages spinal extension and increased lumbar lordosis, which may aggravate the pain. Spinal flexion exercises should be emphasized because they reduce the lordosis and the stress applied to the lumbar spine. Strengthening of the abdominal muscles would increase lumbar spine support. Inclined treadmill testing causes longer walking times, due to spinal flexion.

A patient with adhesive capsulitis is referred to the clinic for physical therapy. In combination with a physical therapy program that consists of range of motion and stretching exercises, deep heat is one of the modalities to improve shoulder mobility. Which of the following modalities is the most ideal for the patient? A. Shortwave diathermy B. Microwave diathermy

C. Electric heating pad D. Ultrasound

Answer: D – Ultrasound is the most appropriate heat treatment for the patient. This modality is ideal for providing deep heat to large joints, such as the shoulder, as a standing wave is produced by the curved reflection of the glenoid. This effect concentrates the energy of heat at the articular surface of the shoulder joint. Option A is the ideal method for large-area deep heating. Option B is recommended for muscle contractures. Option C is a superficial heat technique.

An athlete who is diagnosed with anterior shoulder dislocation is referred for a physical therapy rehabilitation program. The program is designed to be completed in three phases. Which of the following interventions is the least appropriate during phase 1? A. Addition of isokinetic strengthening exercises. B. Active shoulder internal and external rotation exercises with

rubber tubing. C. Active shoulder flexion with available range of motion. D. Shoulder shrug exercise without traction in the glenohumeral joint between repetitions.

Answer: A – Isokinetic strengthening exercises may be added during the second phase of the therapy program. During the first two phases, placing undue stress on the anterior joint capsule must be prevented. Isokinetic test is suggested during the second phase; the shoulder must be free from pain and significant swelling.

A physical therapist is treating a patient who was recently admitted to a facility due to stroke. The therapist attempts to improve the patient’s motor control and balance and normalize the muscle tone through a variety of techniques. Brunnstrom approach is included in the program. This approach focuses on which of the following techniques? A. Tonic reflexes to elicit muscle contractions.

B. Activation of normal righting and equilibrium reactions. C. Diagonal patterns of movement. D. Elimination of unnecessary muscle activity.

Answer: A – Brunnstrom approach utilizes tonic reflexes to elicit muscle contractions. This approach also uses associated reactions, voice commands, resistance, and associated reactions for muscle contraction stimulation. Option B is implemented during neurodevelopmental treatment or Bobath approach. Option C is one of the techniques used by proprioceptive neuromuscular facilitation. Option D is utilized by the motor relearning program.



A professional athlete who had a recent meniscal repair is referred to the clinic for physical therapy. Although the process is mainly dependent on the patient’s physical health and condition after the surgery, the exercise program consists of three phases. The exercise regimen during the second phase is based on which of the following goals of therapy?

A. Regain balance and coordination. B. Regain full knee motion and strength. C. Weight bearing without discomfort. D. Return to normal physical activity.

Answer: B – The second phase of the therapy consists of exercises that aim to regain full knee motion and strength. Bicycle exercises on a stationary bike and short arc quadriceps extensions may be initiated. The first phase consists of exercises that aim to regain balance and coordination. Isometric quadriceps and straight leg raising exercises may be started in the sitting or supine position. Phase three includes exercises that aim to help the patient return to normal physical activity.



A patient is scheduled for decompressive lumbar laminectomy due to significant lumbar radiculopathy. After the surgery, the physical therapist encourages the following activities except: A. Basic mobility activities B. Immobilization C. Proper body mechanics D. Back safety techniques

Answer: B – After the surgery, it is essential that the patient is encouraged to ambulate as soon as possible. Basic mobility exercises, proper body mechanics, and back safety exercises are also encouraged. A short course of physical therapy is recommended. The program consists of lower back and abdominal muscle strengthening exercises.

Traction is the drawing and pulling or the application of forces to the body to stretch a specific part. It is also used to separate two or more parts. In physical therapy, the use of traction is limited to which of the following? I. Cervical region of the spine II. Thoracic region of the spine III. Lumbar region of the spine

A. I and II B. II and III C. I and III D. I, II, and III

Answer: C – In physical therapy, the use of traction is limited to the cervical or lumbar spine. The goal of the therapy is to alleviate pain in or coming from these areas. Traction is also used in the treatment of fractures.

A patient with a history of type 2 diabetes mellitus complains of deep pain in the feet and legs. Muscle cramps and numbness are reported. In addition to non-pharmacologic interventions, which of the following medications is most likely prescribed by the physician first to treat the pain? A. Meloxicam B. Duloxetine C. Carbamazepine

D. Cortisone

Answer: B – Duloxetine, a serotoninnorepinephrine reuptake inhibitor, is the first-line treatment for diabetic neuropathy. Option A is an analgesic commonly prescribed in patients with neuropathic pains. Option C is the first-line treatment for trigeminal neuralgia. Steroids are may be prescribed to patients with neuropathic pains to decrease endoneurial edema.

A physical therapist is treating a patient who has had a stroke. On assessment, shoulder subluxation and pain in the affected arm are noted. Which of the following interventions is least likely implemented to manage the affected shoulder? A. Active weight bearing exercises B. Passive limb physiotherapy C. Functional electrical stimulation

D. Reciprocal pulley

Answer: D – Reciprocal pulley was reported to increase the risk of developing shoulder pain. Option A is used to lengthen or inhibit spastic muscles and facilitate inactive muscles. Passive limb physiotherapy involves maintenance of full painfree range of movement without causing joint trauma. Functional electrical stimulation is used for the treatment of shoulder subluxation, restoration of function in the upper and lower limbs, and treatment of spasticity.

A patient who is diagnosed with lumbar disk herniation is referred to the clinic for a therapy program. Review of patient record reveals signs of lumbar radiculopathy. The physical therapist is correct to include which of the following activities to the program? A. Bed rest for the first 5 days. B. Encourage flexion bias. C. Limit sport-specific pylometrics

D. Maintenance of neutral spine position.

Answer: D – Maintenance of neutral spine position is encouraged. This position helps reduce intradiskal pressure. Bed rest may be recommended 1 to 2 days after an acute episode. Extension bias is encouraged. This position supports balanced segmental force distribution between the disk and zygapophyseal joints. Sport-specific pylometrics can be included to help the lumbar spine maintain a neutral position.

The child’s left leg is shorter than the right in supine position with the hips and the knees flexed. Assessment findings indicate that the child has development dysplasia of the hip. Before the surgery, traction is ordered to the patient. Which of the following is the most appropriate traction for the patient? A. Buck’s extension traction B. Bryant’s traction C. Russell traction

D. 90-90 traction

Answer: B – Bryant’s traction is the most appropriate traction for a patient with developmental dysplasia of the hip. This traction helps keep the top of the femur in the hip socket. Buck’s extension traction is more appropriate for hip fractures. Russel traction is indicated to patients with a femur fracture. The 90-90 traction is indicated to children with a fracture in the femoral shaft.

A physical therapist is reviewing the indications for hydrogel dressings. This type of dressing material is considered if the goal of wound care management includes the following except: A. Rehydration of dry or necrotic tissue B. Wound debridement by autolysis C. Fill in a dry or moist cavity with an entry too narrow to pack

D. Establish environment

a

moist

wound

Answer: D – Establishing a moist wound environment is not an indication for hydrogel dressings. This type of material is mainly used to rehydrate dry wounds. Because hydrogel dressings maintain a moist environment, they can also support autolysis. Hydrogel dressings are also used as fillers in cavity wounds.

A male patient complains of pain in the inner aspect of the left knee. Assessment findings include swelling and knee instability when making a cut. An injury involving the medial collateral ligament is suspected. On valgus stress test, a laxity of 7 mm is noted. Which of the following interventions is least likely implemented to the patient? A. Bracing with a short-hinged knee orthosis that blocks 20-degrees of terminal extension but allows full

flexion. B. Initial non-weight bearing on the affected extremity. C. Crutches for the first few days. D. Closed-chain exercises

Answer: B – Initial non-weight bearing on the affected lower extremity is more appropriate for a patient with Grade III MCL injury. A laxity of 7 mm indicates that the patient has Grade II MCL injury. The patient with this type of injury may ambulate with weight-bearing as tolerated.

A physical therapist uses a type of massage that involves striking of the soft tissue with repetitive blows, using the hands in a rhythmic and rapid manner. The therapeutic effect of this massage is caused by the compression of trapped air on impact. This type of massage is: A. Deep friction B. Effleurage C. Pétrissage

D. Tapotement

Answer: D – Tapotement is a percussion-oriented massage that utilizes the compression of trapped air on impact to bring about therapeutic effects. Deep friction applies shear forces to underlying tissues. Effleurage uses light strokes across the skin over the skeletal muscle that is being treated. Pétrissage involves compression of skin and muscle between the fingers and thumb.

Which of the following treatment approaches mainly focuses on the execution of diagonal, spiraled patterned movements to improve muscle strength and enhance neuromuscular control? A. Neurodevelopmental treatment B. Proprioceptive neuromuscular facilitation C. Brunnstrom’s movement D. Muscle re-education approach



Answer: B – Proprioceptive neuromuscular facilitation is based on spiral-diagonal movement. This technique involves four neurophysiological mechanisms: reflexes, resistance, irradiation, and successive induction. Neurodevelopmental treatment utilizes postural control as the foundation on which patients develop skills. Brunnstrom’s movement encourages the development of flexor and extensor synergies during early recovery. Muscle re-education mainly

develops coordinated movements, beginning with learning to control individual muscles on a cognitive level.

A patient with a significant limitation in range of motion of the left shoulder is referred to the clinic for rehabilitation. The physical therapist uses a technique that passively moves the affected extremity through an antagonistic pattern until resistance is felt. At this point, the patient is told to isotonically contract the antagonistic muscles in a diagonal pattern as the therapist applies resistance. After this contraction, the patient is told to relax, and after interval the therapist repeats the technique from the start. The proprioceptive neuromuscular

facilitation technique used is: A. Repeated contraction B. Hold-relax contract C. Hold-relax D. Contract-relax

Answer: D – The physical therapist uses the contract-relax technique. Repeated contraction and hold-relax are the other two techniques most commonly used for shoulder injuries. A facilitation pattern may be implemented to enhance flexibility.

A male patient is referred to physical therapy for the treatment of anterior glenohumeral instability. During the acute phase, the physical therapist is correct to include which of the following interventions? A. Active range-of-motion exercises concentrating on the joint capsule B. Pendelum exercises C. High-speed, high-energy strengthening exercises

D. Isotonic dumbbell resistance exercises with the supraspinatus, internal rotators, and external rotators

Answer: B – Pendelum exercises are initiated during the acute phase of rehabilitation. The goals of treatment for the acute phase are inflammation and pain relief, re-establishment of non-painful range of motion, and normalization of arthrokinematics of the shoulder. Other exercises initiated are symptom-limited activeassistive ROM and isometric strengthening exercises. Options A and C are more appropriate during the recovery stage. Option C is initiated during maintenance stage.

Western massage is the main type of massage practiced in the United States. One form of Western massage is the petrissage. A physical therapist is demonstrating the correct way of performing this massage if which of the following is observed? A. The physical therapist compresses the underlying skin and muscle between the fingers and thumb of one hand. B. The physical therapist’s hands

glide across the skin and over the muscle being treated. C. The pressure is applied with the ball of the physical therapist’s thumb to the patient’s skin and muscle. D. The therapist strikes the tissue with repetitive blows, using the hands in a rhythmic and rapid manner.

Answer: A – Pétrissage is done by compressing the underlying skin and muscle between the fingers and thumb of one or two hands. The effects of this massage form are compression, release of soft tissues and reactive blood flow. Option B is effleurage. Option C is deep friction. Option D is tapotement.

A physical therapist is evaluating a patient who is referred to the clinic for a cervical spondylosis treatment program. The therapist understands that which of the following is the mainstay of conservative treatment for patients with cervical spondylosis? A. Immobilization of the cervical spine B. Cervical exercises C. Application of heat

D. Isometric strengthening

exercises

for

Answer: A – Immobilization of the cervical spine is the mainstay of conservative treatment for cervical spondylosis. Immobilization can limit the neck motion, which reduces nerve irritation. Manual traction is the most commonly used technique. Options B, C, and D are also implemented, but they are not considered as the mainstay of conservative treatment for cervical spondylosis.

A physical therapist uses a proprioceptive neuromuscular facilitation technique of repeated contraction. This specific technique uses two basic patterns for the shoulder and hip. For the shoulder motions, D1 is referred to as: A. Flexion-abduction-external rotation B. Flexion-adduction-external rotation C. Extension adduction-internal

rotation D. Extension-abduction-internal rotation

Answer: B – D1 refers to flexionadduction-external rotation of the shoulder. Option A refers to D2. Options C and D are reciprocal shoulder patterns.

A patient who is recently diagnosed with glenohumeral dislocation is referred to the clinic for physical therapy. Which of the following interventions is the most appropriate for the patient during the acute phase? A. Elbow, wrist, and hand range of motion during immobilization B. Shoulder pendulum exercises C. Overhead pulley system for the shoulder

D. Rotator exercises

cuff

strengthening

Answer: A – Elbow, wrist, and hand range of motion during immobilization with a sling is recommended during the acute phase. This intervention prevents joint stiffness. Options B and C are initiated during the recovery stage. Option D is initiated during the maintenance stage.

A patient diagnosed with cervical radiculopathy is referred for a rehabilitation program. During the acute phase of the disease, the physical therapist understands that which intervention is the least appropriate for the patient? A. Manual traction B. Use of a semi-hard cervical collar C. Epidural corticosteroids D. Stretching exercises of the neck

and shoulder girdle muscles

Answer: D – Shoulder girdle and neck flexibility exercises are initiated during the recovery stage. During the acute phase of the disease, the interventions focus on reducing the pain and inflammation. The patient is managed by icing, administration of nonsteroidal anti-inflammatory medications or transluminal or transforaminal corticosteroids, and manual traction. The recovery stage includes interventions that aim to restore full range of motion and flexibility of the neck and shoulder girdle muscles.



A physical therapist is treating a patient who has a granulating small wound with mild exudate in the left foot. The wound does not need frequent review. Which of the following dressings would be most appropriate for this wound? A. Calcium alginate B. Polyester fabric with adhesive backing C. Polyurethane foam dressing

D. Adhesive tape

Answer: C – The most appropriate dressing is the polyurethane foam dressing. Foam dressings are used for wounds with mild to moderate exudate that do not require frequent review. Calcium alginate is used for highly exudative wounds. Polyester fabric with adhesive backing is used for wounds with mild to moderate exudate that do not need frequent review. Adhesive tapes are used for exudative or large wounds.

A female patient with a bunion in the left foot complains of severe foot pain that limits her everyday activities, such as walking. Wearing of athletic shoes does not resolve difficulty of walking. Medications and rest do not relieve toe inflammation. Assessment findings include hallux valgus angle of 18degrees and intermetatarsal angle of 10-degrees. Surgery is indicated. Which of the following is most likely considered for the patient?

A. First-metatarsal osteotomy B. Arthrodesis C. Distal soft-tissue reconstruction D. Resection arthroplasty

Answer: C – Distal soft-tissue reconstruction is indicated for patients with mild subluxations. An HVA less than 20-degrees and an IMA less than 11-degrees indicate mild subluxation. The procedure consists of medial eminence excision and medial suture of tear in a capsule. Option A is recommended for patients with moderate subluxations. Option B is indicated for moderate to severe subluxations. Arthrodesis is used for patients with severe arthritis and severe bunions.

A physical therapist is planning the treatment program for a patient with cervical radiculopathy. Which of the following is the most appropriate intervention during the recovery stage of the disease? A. Manual traction B. Maintenance of proper head and neck positioning in everyday activity C. Initiation of resistive cervical stabilization exercises in various planes before isometric strengthening

D. Strengthening of the scapular stabilizing muscles.

Answer: D – Strengthening of the scapular stabilizing muscles are initiated during the recovery stage. After the pain and inflammation are resolved, the therapist can begin with isometric cervical muscle strengthening. Muscle strengthening can then progress to manual resistive exercises that aim to achieve cervical stabilization. Manual traction is most appropriate during the acute stage. Maintenance of proper head and neck positioning in everyday activity is most appropriate during the maintenance stage.



A patient with type IV SLAP lesion had a surgery that involved resection of the affected area of the tendon. After the surgery, the patient is immediately referred for therapy. The physical therapist includes which of the following interventions? A. Active-assisted forward flexion with the use of pulleys B. Active-assisted external rotation movements C. Wrist and elbow active range of

motion exercises D. Endurance training with the use of isokinetic exercises

Answer: C – After the surgery, the patient may initiate wrist and elbow active range of motion exercises. Option A is more appropriate three to six weeks after the surgery. Option B is included in the program six to eight weeks after the surgery. Option D is initiated ten to 14 weeks after the procedure.

A 5-year-old girl swings each leg outward as she walks to take a step without striking the limb in contact with the ground with the moving limb. The child’s knees touch but the ankles are apart when standing. Which of the following interventions is the most appropriate for the patient? A. No treatment is recommended B. Application of extraperiostal 2hole plate

C. Bracing D. Shoe modifications

Answer: A – Genu valgum is usually seen in children between 2 and 6 years. The parents should be educated what to expect and when to seek treatment. If the child continues to demonstrate knocked knees at age 7, night bracing may be indicated. Shoe modifications are not recommended at this time. Surgery may be considered if the condition occurs beyond childhood and in cases of severe separation between the ankles.

After pain and inflammation are controlled, the therapy of a patient with cervical radiculopathy is progressed to the restoration of full range of motion and flexibility of the neck and shoulder girdle muscles. The following exercises are implemented during the recovery stage except: A. Isometric strengthening of the cervical muscles in a single plane. B. Progressive isotonic activity of

the scapular stabilizing muscles. C. Closed kinetic chain exercises focusing on the shoulder girdle muscles. D. Isokinetic exercises of the neck and upper extremities.

Answer: D - Isokinetic exercises of the neck and upper extremities are not functional and are not recommended for strengthening the cervical muscles. Before beginning complex activities involving multiple muscles, it is recommended to implement isolated strengthening of individual muscles affected by the radiculopathy.

A patient sustained direct trauma to the shoulder, causing dislocation of the anterior glenohumeral joint. Which of the following is the most appropriate initial treatment for patients with glenohumeral dislocation? A. Shoulder immobilization B. Wrist and elbow exercises C. Corticosteroid administration D. Reduction



Answer: D – Reduction is the initial treatment for glenohumeral dislocations. Muscle spasms may develop after a dislocation, and initiating reduction after its development can be difficult. Because of this, reduction should be the priority. Immobilization is initiated after the reduction.

A physical therapist is treating a patient who has a healable wound cavity with large amounts of exudate. The physical therapist manages the wound by coating the dressing with an antiseptic. What is the most appropriate wound dressing for the patient? A. Non-adherent dressing B. Hydrocolloid dressing C. Hydrogel dressing

D. Calcium alginate dressing

Answer: D – Calcium alginate is used to absorb wounds that drain moderate to large amounts of exudate. The other dressings are used to absorb wounds that drain minimal to moderate amounts of exudate. Non-adherent dressings are used for superficial wounds. Dry necrotic wounds are managed by hydrocolloid dressings. Hydrogel dressings are useful for dry, sloughy and necrotic wounds.

A female patient is admitted to the facility for a scheduled total knee replacement. A physical therapist informs the patient of the exercises and activity limitations that she has to follow after the surgery. The physical therapist encourages the patient to perform which of the following activities in the recovery room, if tolerated? A. Keep the affected extremity immobilized.

B. Perform quadriceps sets during a two-minute period until fatigued. C. Walk using a walker or a pair of crutches. D. Perform assisted knee bends until fatigued.

Answer: B – The most appropriate instruction is to encourage the patient to perform quadriceps sets as soon as she is able to perform the exercise. Early postoperative exercises, including straight leg raises, knee straightening exercises and knee bends with support, may be started in the recovery room after the surgery. These exercises may diminish postoperative pain and promote recovery. Walking in short distances may be started in the hospital room. Assisted knee bends are initiated after the patient regains

independence for short distances.

A female athlete who was diagnosed with Grade III medial meniscal injury informs the physician she cannot risk the delay of a possible unsuccessful period of observation. Surgery is indicated to the patient. If the athlete’s meniscal tear involves greater than 50% of the meniscal thickness, which of the following procedures is the most appropriate for the patient? A. Intra-articular reconstruction

B. Partial meniscectomy C. Meniscus transplantation D. Meniscus repair

Answer: D – Meniscus repair is the most appropriate intervention for the patient. This procedure is recommended to meniscal tears greater than 50% of the meniscal thickness, occurring in the vascular region, unstable to arthroscopic probing, and measuring more than 1 cm in length. Option A is considered for anterior cruciate ligament injuries. Option B is the treatment of choice for meniscal tears in the avascular area. Option C requires further investigation.

A 1-month old female is positive for Ortolani and Barlow tests, indicating an unstable hip. The physical therapist expects which of the following orders from the physician? A. Acetabular procedure B. Initiation of traction before closed reduction C. Use of dynamic flexion-abduction orthosis D. Open reduction



Answer: C – A dynamic flexionabduction orthosis is usually ordered to manage unstable hips of infants younger than 6 months. A Pavlik harness is a dynamic flexionabduction orthosis. Option A is recommended to children older than 4 years with acetabular dysplasia. Option B is ordered to infants older than 6 months and to infants who fail to respond to orthosis. Option D is ordered to children older than 2 years.

After assessing the findings obtained from the history, physical examination and diagnostic tests, a male patient is diagnosed with complete spinal injury. Diagnostic tests reveal that the first lumbar vertebra is involved. Considering the bowel function changes resulting from complete spinal cord injuries, which of the following interventions is the most appropriate for the patient? A. Stimulation of the rectum with a

gloved lubricated finger. B. Suppository insertion C. Manual evacuation of the stool D. Micro enema administration

Answer: C – Manual evacuation of the stool is most appropriate intervention for the patient. Injuries involving the first lumbar vertebra and below causes a flaccid bowel with a lax anal sphincter and pelvic floor. Slow stool propulsion and fecal incontinence are likely. Stimulation of the rectum through a gloved lubricated finger, suppository insertion and micro enema administration is more appropriate for patients with injuries to the twelfth thoracic vertebra and above. In this condition, the reflex arc from

the cord to the colon and ano-rectum is intact.

A physical therapist is evaluating a patient who has a history of spinal cord injury that affects the fifth cervical vertebra. As the therapist plans for the patient’s rehabilitation program, the therapist should consider that patients with C5 spinal injury require: A. A ventilator B. A manual wheelchair for long distances on flat surfaces C. Assistance when transferring from

a bed to a wheelchair D. Assistance to feed self with feeding straps during meals

Answer: C – A patient with C5 spinal cord injury requires assistance when transferring from a bed to a wheelchair due to complete paralysis of the body and legs. The patient is able to breathe without a ventilator. An electric chair that can be controlled with a hand control is used for uneven surfaces. The patient has the ability to feed self with feeding straps.

A 25-year-old female delivers a female newborn with talipes equinovarus involving the right foot. Foot cavus is not noted. The physician decides to correct the deformity with Ponseti management. Which of the following interventions is least likely included in the treatment program using the Ponseti management? A. It is started within 7 to 10 days after birth.

B. It involves six or seven plaster cast changes. C. Manipulation consisting of foot abduction is the first element of the management. D. Maintain the affected foot in pronation.

Answer: D – Pronation of the foot is avoided. This position may increase the cavus and encourage the development of new deformity of eversion through the mid- and forefoot, causing a bean-shaped foot. Pronation does not do anything to abduct an adducted and inverted calcaneus bone. The Ponsi method is started after birth. The management program involves weekly manipulations for six weeks. If the deformity is not corrected after six or seven plaster changes, the treatment is likely faulty. If foot cavus is

assessed, it is the first element of the therapy. Without a foot cavus, manipulation is the first step of treatment.

A female patient with trigeminal neuralgia is prescribed to undergo non-pharmacologic and pharmacologic interventions to control and manage the pain. Which of the following medications is most likely prescribed to the patient as it is considered the first-line therapy for trigeminal neuralgia? A. Gabapentin B. Carbamazepine

C. Phenytoin D. Baclofen

Answer: B – Carbamazepine is a first-line treatment drug for trigeminal neuralgia. Options A, C, and D are also prescribed to patients with this condition, but they are not usually prescribed during the initial drug treatment. Carbamazepine works by inhibiting the sodium channel activity, which reduces the excitability of neurons.

A patient demonstrating foot drop is diagnosed with peroneal neuropathy due to compression at the fibular head. The foot drop is secondary to lumbar disk herniation. Which of the following interventions is least likely ordered for the patient? A. Knee-ankle-foot orthotics B. Erythropoietin in 3 doses of 5000 U/kg. C. Discectomy

D. Gait training

Answer: A – Foot drop is usually managed by knee-ankle-foot orthotics. AFO supports dorsiflexion during the swing phase and medial and lateral stability at the ankle during stance. AFOs are only recommended if the foot is able to achieve plantigrade. Erythropoietin may be ordered to facilitate cell survival. Discectomy may be ordered if the foot drop is caused by lumbar disk herniation. Gait correction is incorporated into the treatment program.

A patient complains of sharp pain in the knee. Swelling and local tenderness are noted during assessment. Examination reveals that the pain is increased with knee motion. Which of the following is the most appropriate intervention for the patient? A. Application of heat 18 hours after the injury. B. Weight bearing with the injured extremity up to the point of pain.

C. Elevation of the affected extremity above the heart. D. Application of compression dressing from proximal to distal.

Answer: C – The patient is instructed to elevate the affected extremity above the heart. This intervention decreases the edema and promotes venous return. Heat is applied 24 to 48 hours after the injury. Weight bearing is avoided during the acute stage. The patient is instructed to rest the injured ligament to heal. Compression dressing is applied from distal to proximal to reduce edema.

A patient with a fractured tibia complains of pain in the injured extremity. After the pain is localized, which of the following interventions is least likely to relieve pain that is secondary to a fractured tibia? A. Intake of analgesics. B. Change of positions to relieve pressure. C. Prevention of range of motion exercises and isometric muscle contractions.

D. Elevation of the involved extremity.

Answer: C – The patient is encouraged to perform isometric contractions by wiggling the toes. This intervention reduces the edema and improves venous return. Options A, B, and D are recommended to relieve pain secondary to a fracture.

A physical therapist is treating a patient who had a lumbar disk surgery a day ago. The therapist ensures safety and prevention of complications during physical treatment by demonstrating which of the following actions? A. When turning and repositioning, the head of bed is raised to 30degrees. B. When positioning the patient, a pillow is placed under the head with

the knees extended. C. When assisting a patient out of bed, place the bed in flat position as the patient lies on the side. D. When turning the patient, the patient is turned as a unit without twisting the back.

Answer: D – When turning, the patient should be moved as a unit without twisting the back. When turning and repositioning, the bed should be in flat position. When assisting the patient out of the bed, the head of bed should be raised. A pillow must be placed under the head with the knees slightly flexed.

A patient complains of swelling and pain in the left calf. The patient is positive for Homan’s sign. On palpation, the affected area is warm to touch and tender. A diagnosis of deep vein thrombosis is made. Which of the following interventions is the most effective in the treatment of deep vein thrombosis? A. Massage therapy. B. Application of cold compress to affected extremity at least four times

daily. C. Elevation of the legs 10 to 20 minutes every few hours. D. Ambulation.

Answer: C – Elevation of the legs 10 to 20 minutes every few hours is the most effective intervention. Massage is contraindicated to patients with deep vein thrombosis as it can cause the clot to break free from the vein. Warmth promotes vasodilation. Bed rest is maintained because using the leg muscles during walking can aggravate the inflammatory process.

A patient complains of aching pain in the left leg, which becomes more severe in intensity at night. Standing relieves the pain. On assessment, the affected extremity feels cool and appears red in dependent position. A diagnosis of thromboangitis obliterans is made. Which of the following interventions is the most important for this condition? A. Vasodilator therapy B. Prevention of injuries on the

upper and lower extremities. C. Smoking cessation D. Keeping the extremities warm

Answer: C – Smoking cessation is the most important intervention for the patient. This condition is caused by occlusion of small and mid-size peripheral arteries. Although smoking cessation does not cure the disease, it may slow down the extension of clots. Smoking further increases the risk for gangrene and ulcerations. Options A, B, and D are included to the treatment program, but they are not the most important intervention for patients with thromboangitis obliterans.

A female patient reports that her fingers turn blue when exposed to cold temperatures. The affected fingers turn white and then red with re-warming. A diagnosis of Raynaud’s disease is given. Which of the following is the least appropriate intervention for the patient? A. Keep her hands warm by wearing gloves when exposed to cold temperatures.

B. Keep her hands protected by wearing kitchen gloves when exposed to hot items. C. Swing her arms back and forth during onset of pain. D. Elevate the affected extremity for 10 to 20 minutes every day every few hours.

Answer: D – Elevation of the affected extremity is not appropriate for a patient with Raynaud’s disease. Protection of extremities from injuries is a priority. Swinging the arms back and forth during the onset of pain can stop the painful episode.

Each form of massage uses a particular rhythm, rate, pressure, direction, and technique to bring about its therapeutic effects. A physical therapist understands that the different variables of massage are mainly controlled by the practitioner. Which of the following massage techniques correctly achieves the therapist’s treatment goal? A. The therapist uses circular motions of shearing forces to treat muscle spasms.

B. The therapist uses a heavier pressure to produce relaxation and sedation. C. The therapist uses motions that are kept parallel to the muscle fibers to treat the muscles. D. The therapist uses kneading massage to breakdown adhesions.

Answer: C – When treating a muscle, the motions should be kept parallel to the muscle fibers. Circular motions of shearing forces are used to breakdown adhesions. A lighter pressure is used to produce relaxation and sedation. Kneading massage decreases edema.

Safety and Professional Role, and Teaching, Learning, Research

Maintaining good communication skills is essential throughout the rehabilitation process, as it provides compliance to therapy and encouragement. Which of the following actions by a physical therapist does not demonstrate good communication technique? A. Standing in front of a patient who is sitting in a wheelchair to provide discharge instructions. B. Making an eye contact when

providing home instructions. C. Providing the patient with written exercise descriptions before discharge. D. Repeating the patient’s uncertainties.

Answer: A – When talking to a patient, it is recommended to stay in front of the patient at the same eye level to encourage communication. Maintaining good eye contact facilitates good communication. Exercise descriptions ensure better recall and compliance. Repeating the patient’s uncertainties is an active listening skill; it demonstrates interest and concern.

Transcutaneous electrical nerve stimulation is contraindicated to patients with a demand-type of pacemaker and to women who are pregnant. In addition to these, the TENS should also be placed in the right areas to prevent injuries and complications. TENS should not be placed in the following areas except: A. Carotid sinuses B. Anterior neck C. Area of sensory impairment

D. On the skin over the painful area

Answer: D – The electrodes are placed on the skin over the painful area. The electrodes must not be placed on the carotid sinuses to reduce the risk of hypotension. Electrodes placed on the anterior neck may cause laryngospasm. When electrodes are placed over an area of sensory impairment, it increases the patient’s risk for burns.

A physical therapist is conducting a teaching session to a group of elderly patients in a community center. The therapist understands that a significant number of musculoskeletal disorders in the elderly are caused by falls. Which of the following prevents falls in the elderly? A. Decrease the intensity and duration of exercise. B. Wear shoes with thin, non-slip

soles. C. Use small throw rugs at home. D. Use a step stool when reaching things.

Answer: B – Wearing thin, non-slip soles can prevent falls. The elderly are encouraged to maintain muscle strength and promote balance and coordination. Small throw rugs should be avoided. It is recommended to place frequently used items within easy reach to avoid step stools.

A physical therapist is treating a patient with a clean and dry wound with minimal exudate in the left lower extremity. Which of the following dressings are contraindicated for this wound? I. Hydrocolloid dressings II. Hydrogels dressings III. Foam dressings IV. Calcium alginate dressings

A. I, II, IV B. III, IV C. I, II D. II, III

Answer: B – Foam dressings and calcium alginates are contraindicated in dry wounds. Foam dressings are contraindicated in dry wounds where moisture absorbency is not required for balance. Alginates are highly absorbent, as they could dehydrate the wound and cause delayed healing. Hydrocolloid and hydrogel dressings are useful for clean and dry wounds with minimal exudate.

A patient had an above-the-knee amputation. During the postoperative period, which of the following interventions promotes healing and preventions complications of amputations? A. Elevate the stump during the first 24 hours. B. Massage the skin toward the suture line.

C. Position the patient in prone position during the first 24 to 48 hours. D. Promote internal or external rotation of the limb.

Answer: B – Massage the skin toward the suture line to promote good circulation. The foot of the bed is elevated during the first 24 hours. The stump itself is not elevated as it can cause flexion contracture. The patient is then placed in prone position 24 to 48 hours after the surgery. Internal or external rotation must be avoided; a rolled towel along the outside of the thigh can be used to prevent rotation.

A male patient who had a total knee replacement is ordered to use a pickup walker with four solid prongs to support balance and walking. Which of the following instructions given by a physical therapist is the most appropriate for the patient when ambulating using a walker? A. Step all the way to the front bar of the walker. B. Using both hands, grip the top of the walker, and walk into it.

C. Step off on the uninjured leg and complete the step with the injured leg. D. Take bigger steps when turning.

Answer: B – The patient grips the top of the walker with both hands and walks into it. The patient should then step off on the injured leg and complete the step with the good leg. When making a turn, the patient is advised to take smaller steps.

A 30-year-old white female is referred for therapy after a fractured tibia-fibula. The patient’s record indicates that the patient has a family history of osteoporosis, making her at risk for the same bone disorder during advanced age. To prevent osteoporosis, the patient is informed to: A. Consume green and leafy vegetables. B. Decrease physical activity.

C. Exercise at least 15 minutes twice to thrice a week. D. Use a soft mattress.

Answer: A – The patient is recommended to consume green and leafy vegetables. These food items are high in calcium. The patient is encouraged to maintain an exercise program at least 20 minutes four or more times a week. A firm bed mattress is recommended to maintain posture and prevent back pain.

A female patient with severe acute pain in the back is referred for physical therapy. The history of the patient reveals light activity at work. The patient is required to perform occasional lifting and bending. The physical therapist informs the patient that she can return to unmodified work in 3 days. The therapist is correct in giving which of the following instructions to the patient before discharge? A. Avoid lifting more than 25

pounds. B. Have a five-minute break from sitting or standing every 30 minutes. C. Driving a heavy vehicle must be limited to four hours a day. D. Driving a car must be limited to six hours a day.

Answer: B – A patient with severe acute pain in the back is instructed to have a five-minute break from sitting or standing every 30 minutes. Options A, C, and D are recommended for patients with heavy activity levels at work.

A patient who was admitted due to deep vein thrombosis is ready for discharge. The physical therapist gives the patient a list of home care instructions that may help prevent the occurrence of another episode of deep vein thrombosis. Which of the following instructions is not included in the list? A. Active range of motion exercises every 8 hours. B. Maintain a progressive walking

program. C. Elevate the legs, with the knees slightly flexed. D. Wear anti-embolic stockings at all times, but remove them when bathing and sleeping.

Answer: D – The patient is instructed to remove anti-embolic stockings 30 to 60 minutes a day to allow assessment of the tissues and to encourage restoration of perfusion of the dermis. An active range of motion exercises when on bed rest maintains joint mobility. Without an acute episode of deep vein thrombosis, a progressive walking program is encouraged. The legs are elevated with the knees flexed; Elevation enhances venous return and knee flexion promotes muscle relaxation.



A male elderly is ordered to use a single-point cane at home. The order is given after the patient complains of weakness in the left leg. The patient asks a physical therapist about the proper of way of climbing and going down the stairs. Which of the following answers made by the therapist is the most inappropriate? A. “Step up on your bad leg first when climbing the stairs.” B. “The cane should be held in the

hand opposite the affected leg.” C. “Put your injured leg first and then your cane on the first step when going down the stairs.” D. “Step down with your injured leg on the first step and then step down on the good leg.”

Answer: C – The patient should first put the cane on the first step, then the injured leg when going down the stairs. The good leg steps down next. The other options are correct instructions.

A physical therapist is treating a patient who has had a stroke. The patient’s verbal communication skills are impaired. When interacting and talking to the patient, the therapist maintains good communication techniques by the following actions except: A. Use short statements and simple questions. B. Stand in front of the patient and speak slowly.

C. Allow adequate time for the patient to respond. D. Use a raised voice when talking to the patient.

Answer: D – Using a raised voice does not facilitate good communication. The physical therapist must talk to the patient using a low-pitched voice. The therapist’s statements and questions should be short and simple to decrease the patient’s frustration. Standing in front of the patient facilitates clarity and understanding. Allowing adequate time for the patient to respond decreases frustrations and motivates the patient to communicate.

A male elderly with osteoarthritis of the right knee is ordered to use a single-point cane to improve balance when walking. A physical therapist provides the following instructions when ambulating with a single-point cane on a level surface except: A. The affected leg and the cane should swing and strike the ground at the same time. B. Weight is shifted through the arm holding the cane, as needed.

C. Hold the cane in the hand on the affected side. D. Ensure that the unaffected leg fully bears weight on the first step on level surfaces.

Answer: C – The patient should be instructed to hold the cane in the hand on the unaffected side to provide support to the opposite leg. The patient should start by positioning the cane one small stride ahead and then stepping off on the injured leg. This start allows the unaffected leg to fully bear weight on the first step.

Aquatic therapy is the performance of exercises in water to eliminate gravity and to help support the body. This therapy uses water to provide resistance to movements. Aquatic therapy is contraindicated to the following patients except: A. A patient with fever B. A patient with multiple sclerosis C. A patient with bowel and bladder incontinence

D. A patient with a tracheostomy

Answer: B – Aquatic therapy is indicated to a patient with multiple sclerosis. This therapy may help the patient regain muscle strength without causing significant fatigue. Options A, C, and D are contraindications. Patients with infection, uncontrolled blood pressure, open wound, and abnormally low vital capacity are not advised to engage in this therapy.

A physical therapist is providing postural chest drainage to a patient with active tuberculosis. When providing respiratory treatments, the therapist ensures prevention of disease transmission by which of the following actions? A. Wearing a surgical mask. B. Providing the treatment in a private room with negative airflow. C. Limiting exposure time to no more than 30 minutes per day.

D. Instructing the patient to wear a filter mask before transport.

Answer: B – A patient with active tuberculosis must be placed in a private and closed room with a negative airflow. The therapist must wear a filter mask when providing the treatment. Airborne precautions must be maintained. Limiting exposure time is not necessary. The patient is instructed to wear a surgical mask before transport.

A physical therapist is teaching a patient how to sit from a standing position when using a pair of crutches. Which of the following is the right sequence? I. Slowly move the affected leg forward and lower into the chair II. Place weight on the unaffected leg and advance the affected leg forward III. Place both crutches side by side on the unaffected side IV. Hold both handgrips together and reach back for the arm rest with the

other hand A. IV, III, II, I B. III, II, VI, I C. II, III, IV, I D. IV, II, III, I

Answer: C – The patient backs up against a chair until he feels the chair at the back of his legs. The weight is placed on the unaffected leg and the affected leg is advanced forward. The next step is to place the crutches on the unaffected side, and hold the grips as the patient reaches back for the chair using the other hand. Using the armrest and hand grips, the patient moves the affected leg forward and lowers herself into the chair.

A physical therapist is treating an elderly patient with active range of motion exercises and gait training after a total hip replacement. Before the start of the therapy, the patient informs the therapist that she is skipping a session due to pain in the operative site. The physical therapist informs the patient of the risks of skipping the treatment and the importance of maintaining a regular program, but the patient continues to refuse the treatment. Which of the following the next action of the therapist?

A. Leave the room and recommend an increase of pain medication dose. B. Call the patient’s proxy and explain the situation. C. Record the patient’s refusal to treatment. D. Call the physician and report the refusal.

Answer: C – The next action of the physical therapist is to record the patient’s refusal to treatment. The patient has the right to refuse treatment. The therapist must ensure that the risks and benefits are thoroughly explained. Option A may be considered after careful consideration of the other factors. Increase of dose must be supported by well-documented findings. Option B may be considered only if the patient is incapable to make her own medical decisions. Option D is done after documenting the refusal.



A female patient with carpal tunnel syndrome is referred for therapy, which mainly consists of conservative modalities. The physical therapist in-charge of the patient ensures to include which of the following as an effective and consistent conservative treatment for carpal tunnel syndrome? A. Massage B. Aerobic fitness program C. Stretching exercises

D. Wrist-hand orthosis

Answer: D – Wrist-hand orthosis with the wrist kept in neutral position is one of the most effective and consistent conservative treatments for carpal tunnel syndrome. The orthosis is usually used for three to four weeks. Massage does not offer proven benefits for CTS. Options B and C may be included in the program.

A female patient with low back pain is being instructed on how to perform proper body mechanics when doing the activities of daily living. The physical therapist informs the patient that when lifting a box, the patient should follow which recommendation? A. Maintain a narrow base of support, bend at knees, and flex the lower back. B. Maintain a wide base of support,

bend at the waist, and extend the lower back. C. Maintain a wide base of support, flex forward at the waist, and maintain the spine in neutral position. D. Maintain a wide base of support, bend at knees, and maintain the spine in neutral position.

Answer: D – The proper way of lifting is done by maintaining a wide base of support, bending at knees, and maintaining the spine in neutral position. A wide base of support stabilizes the movement. Bending at the knees prevents application of unnecessary pressure to the back. When lifting, the spine should be maintained in neutral position.

A female patient with muscle contractures is being considered for microwave diathermy, a deep heat modality. As the physical therapist reviews the patient’s history, which of the following details of information contraindicates the procedure? A. A recent cardiac pacemaker procedure B. A diagnosis of periostitis C. A history of rheumatoid arthritis

D. A diagnosis of bursitis

Answer: A – The procedure is contraindicated to a patient who has a history of cardiac pacemaker surgery. Bursitis, rheumatoid arthritis, and periostitis, are managed by microwave diathermy.

Cryotherapy, also known as cold therapy, makes use of ice or cold applications for the treatment of certain musculoskeletal disorders or injuries. The devices and substances utilized cause the withdrawal of heat. This modality is contraindicated to patients with the following conditions except: A. Hypertension B. Raynaud’s disease C. Upper motor neuron spasticity

D. Rheumatoid arthritis

Answer: C – Cryotherapy is indicated to patients with upper motor neuron spasticity. Raynaud’s, rheumatoid arthritis, and hypertension are contraindications to cryotherapy. Cold can cause vasoconstriction, which can aggravate Raynaud’s disease and hypertension. Cold can increase tissue stiffness, which may aggravate rheumatoid arthritis.

The physical therapist prepares a teaching plan for a patient who was admitted due to a cervical injury. The patient is to be discharged with a halo fixation device with jacket. Which of the following instructions is correctly included in the teaching session? A. Clean the pin site every day using clean techniques. B. Eat foods high in carbohydrates and potassium.

C. Maintain tightness of the jacket by ensuring that two fingers are placed between the skin and jacket. D. Showers are prohibited.

Answer: D –The patient should be informed that showers are prohibited. Sponge baths or tub baths are allowed. The pin site should be cleaned every day using aseptic techniques. The patient is encouraged to eat foods high in protein and potassium to facilitate bone healing. The jacket is kept tight by ensuring that only one finger is placed between the skin and the jacket.

A patient with left-brain damage caused by stroke is on the post-acute phase of recovery. The patient’s primary health givers are given which of the following home instructions to ensure good nutrition during motor and sensory recovery? A. Assess the gag reflex on arising. B. Provide liquids during meals. C. Place food in the back of the mouth on the unaffected side.

D. When eating, position the patient in sitting position with the neck slightly extended.

Answer: C – The health givers should be instructed to place food in the back of the mouth on the unaffected side to avoid food trapping in the affected cheek. The gag reflex should be assessed before meals. Soft and semi-solid foods are more tolerated than liquids. When eating, the patient should be placed in sitting position with the neck slightly flexed to facilitate swallowing.

Direct-thrust, a direct manipulation technique, requires well-localized and specifically-directed forces. This technique is contraindicated to the following patients except: A. A patient with spinal deformity B. A patient with osteomalacia C. A patient on systematic anticoagulation treatment D. A patient with atlanto-occipital dysfunction



Answer: D – Atlanto-occipital dysfunction is an indication for direct-thrust techniques. Spinal deformity, osteomalacia, systemic anticoagulation therapy, severe diabetes, congenital joint laxity, osteoporosis severe type, degenerative joint disease, and open wounds are contraindications to the therapy.

A long-leg circular plaster cast is applied to a 12-year-old with a tibiafibula fracture. Before discharge, the physical therapist teaches the child on how to take care of the cast. The therapist is correct to instruct which of the following as a cast-care intervention? A. Keep the cast and extremity flat on bed. B. Petal the cast. C. Use a blow dryer on a hot air

setting to relieve itching. D. Allow the cast to dry in 24 hours.

Answer: B – The child is instructed to petal the cast to prevent crumbling. The casted extremity must be elevated. A blow dryer on cool setting is used to relieve itching. A plaster cast may dry up to 48 hours.

A female patient with musculoskeletal pain is referred to the clinic for physical therapy. The therapist plans to include transcutaneous electrical nerve stimulation application as one of the modalities for the patient. Which of the following details of information in the patient record makes TENS a contraindicated modality to the patient? A. A diagnosis of diabetes neuropathy.

B. A diagnosis of spinal cord injury. C. A history of angina pectoris. D. Use of a demand-type of pacemaker.

Answer: D – Patients with a pacemaker are contraindicated for the use of TENS because the stimulus output of TENS may stimulate or inhibit the pacemaker. Women who are pregnant are not recommended to have TENS as it induces labor. Options A, B and C are indications for TENS.

A male patient on therapy has stage II Alzheimer’s disease. In this stage, the patient’s memory deficits become more evident. The patient is less than able to behave spontaneously. Recognizing the increasing episodes of confusion, the therapist implements which of following interventions to manage chronic confusion? A. The physical therapist begins each session by asking the patient to identify the therapist’s name.

B. The physical therapists avoid using questions that require a yes or no answer. C. Provide boundaries by placing red or yellow tapes on the floor. D. Stimulate the patient’s senses by music and lights.

Answer: C – Creating boundaries can help the patient stay within the safe areas. The physical therapist should begin each session by introducing self. Questions must be limited to those that require a yes or no answer. Stimuli must be kept to a minimum.

A patient with Alzheimer’s disease has an increased risk for injuries due to cognitive impairments. The main health giver of the patient should be instructed to perform which of the following activities at home to ensure safety of a patient with cognitive impairments? A. Increase daytime lighting in dark areas. B. Implement exercises that increase strength and balance.

C. Keep traffic areas at home free from clutter. D. Fence the yard with a locked gate.

Answer: D – Fencing the yard with a locked gate prevents wandering. Options A, B, and C are interventions that decrease the risk of falls. Fencing the yard focuses on ensuring safety for a patient with cognitive impairments.

A female patient with multiple sclerosis is seen by the physical therapist for an initial assessment. The patient has significant fatigue. The patient reports decreased capacity for physical and mental work. After assessing the degree of fatigue, the therapist instructs the patient to do which of the following to conserve energy? A. Encourage cold water baths. B. Instruct the patient to keep up

with the listed activities scheduled for the day to effectively manage the workload. C. Encourage continuous work in the morning; begin a long break in the afternoon. D. Schedule activities that require more energy in the morning.

Answer: D – The patient is instructed to perform more difficult tasks in the morning. The normal rhythm of the body facilitates greater energy in the morning. Extreme temperatures are avoided because they can aggravate the symptoms. The patient is taught to pace her activities. When the schedule is rigid, it can cause more stress to the patient. Short periods of rest between activities may help replenish reserves of energy.

An elderly has stage II Parkinson’s disease. Impaired righting reflexes are noted on assessment. A team of health care providers must work together to manage a patient with Parkinson’s disease. The physical therapist is mainly tasked to promote which of the following activities? A. Adaptation to changing abilities essential for work. B. Domestic activities C. Mobility

D. Effective swallowing

Answer: C – In Parkinson’s disease, the physical therapist is mainly tasked to promote mobility and posture skills. Options A and B are managed by occupational therapists. Option D is managed by speech therapists.

A patient with myasthenia gravis would usually develop weakening of the laryngeal and pharyngeal muscles involved with swallowing. To ensure safety and good nutrition, the physical therapist educates the patient and his or her main health givers to do which of the following? A. Eat with bigger bites of food. B. Schedule meals after an activity. C. Health giver gives cues when the patient is eating.

D. The health giver should learn how to perform cardiopulmonary resuscitative properly.

Answer: C – Giving cues encourages focus, which may enhance swallowing. The patient is instructed to eat slowly with smaller bites of food. The mealtimes should be scheduled around the times when the patient is well-rested. The health giver should learn Heimlich maneuver as the patient is at risk of choking.

A physical therapist is treating an elderly patient who is immobilized due to left-sided muscle weakness. The patient spends most of the day in supine position, which increases the risk for pressure ulcers. Assessment and care in the following bony prominences when caring for a patient in prolonged supine position include the following except: A. Greater trochanter B. Occipital tuberosity

C. Vertebral spinous processes D. Malleolus of fibula and tibia

Answer: D – The malleolus of fibula and tibia is at risk for pressure ulcers in side-lying position. In supine position, the greater trochanter, occipital tuberosity, and vertebral spinous processes are at risk for pressure ulcers.

Positioning of a patient who had a stroke is significantly important for the patient recovery process. Correct postures encourage joint alignment, prevent contractures and injuries, and promote patient’s comfort. In supine position, a patient who has a left hemiplagia must have: A. The head and neck in slight extension. B. A small pillow under the cervical spine.

C. The wrists in slight flexion. D. The affected lower extremity on a pillow in neutral position.

Answer: D – The affected lower extremity must be on a pillow in neutral position. A small rolled towel must be placed beneath the knee. The head and neck must be in slight flexion. A small pillow must be placed underneath the affected scapula. The wrists must be in neutral position.

A physical therapist must maintain proper body mechanics at all times to prevent injuries, especially in the back muscles. A therapist lifting a load is demonstrating proper body mechanics if which of the following actions is noted? A. The therapist keeps the load as far she can from her body. B. The therapist inhales as she performs the actual lift. C. The therapist forward flexes her

trunk when lifting. D. The therapist uses her legs during the actual lift.

Answer: D – The physical therapist should use her legs during the actual lift. The power should come from the large muscle groups of the lower extremities and not from the back muscles. When lifting, the load should be kept close to the therapist’s body. Exhaling during the actual lift decreases a Valsalva maneuver or straining. The therapist should keep the spine in neutral position when lifting.

A physical therapist is writing a questionnaire that is to be sent to and answered by patients after discharge. The therapist would like to create open-ended questions in the questionnaire. Which of the following is an example of an openended question? A. “Did the physical therapy sessions help you achieve your recovery goals?” B. “How have your motor and

sensory functions changed after the treatment program?” C. “Did the sessions of exercises help your pain levels?” D. “Did the interventions address your main concerns?”

Answer: B – Asking the patient how her motor and sensory functions have changed after the treatment program is a technique of asking open-ended questions. Open-ended questions avoid limited responses, such as yes or no.

A physical therapist is assessing a female patient with chronic pain for the first time. The history of the patient reveals repeated and chronic injuries, including contusions and fractures. The assessment reveals bruises in different stages of healing on the upper extremities. Domestic violence is suspected. Which of the following questions is the most appropriate? A. “You have signs of physical abuse. Is someone hurting you?”

B. “You have signs of physical abuse. Don’t worry, you are safe here.” C. “Your bruises look painful. Did your partner hit you?” D. “Did you do something to cause your partner to beat you?”

Answer: C – Giving observations and asking direct questions is a communication technique used when dealing with a patient suspected of abuse. Options A and B are not effective because the physical therapist has already given a diagnosis without gathering information from the patient. Asking the patient what she did to cause her partner to beat her is not appropriate.

About Minute Help Press Minute Help Press is building a library of books for people with only minutes to spare. Follow @minutehelp on Twitter to receive the latest information about free and paid publications from Minute Help Press, or visit minutehelpguides.com.

Related Documents


More Documents from "Jaspreet Kaur"